Section 4 Endocrinology

157

SECTION 4 Endocrinology

158

CHAPTER 7

159

Adrenal Insufficiency

160 Case: A 44-year-old man with acute abdominal pain acute, cramping abdominal pain associated withfnausea, vomiting, Heart rate is 130 beats per minute and blood pressure is A 44-year-old man is admitted to the hospital or evaluation of and watery diarrhea. 90/52 mm Hg. Generalized hyperpigmentation is present (Figure 7- 1A). For comparison, the patient provided an old pho ograph (Figure 7-1B). Scattered patches of hypopigmentation are alsotpresent on the trunk. The abdomen is diffusely tender to palpation. The patient first noted skin changes 8 years ago. He also describes

weight loss, fatigue, and episodes of light-headedness over the past diagnosis was made. few years. Several physicians evaluated him over this time, but no Peripheral white blood cell count is 13 K/”L, serum sodium is 126 mEq/L, and serum glucose is 68 mg/dL. Cross-sectional imaging of infectious or inflammatory ileocolitis. Serum,cortisol leveliis 4.4 ”g/dL test is 872 pg/mL (reference range 10-60 pg/mL). Closer review of the the abdomen shows diffuse thickening of the terminal ileum and ascending, transverse, and descending colon consistent with 60 minutes after a 250 ”g injection of synthetic adrenocort cotropic hormone (ACTH). Plasma ACTH level drawn prior to the stimulation abdominal imaging reveals diminutive adrenal glands.

FIGURE 7-1

What is the most likely underlying diagnosis in this patient?

What is adrenal insufficiency? What is the normal hormonal cycle of the hypothalamic-pituitary-adrenal axis?

Adrenal insufficiency is a clinical condition that results from deficient production or action of glucocorticoids, with or without mineralocorticoid or androgen deficiency.1 The hypothalamus produces corticotropin-releasing hormone (CRH), which stimulates the pituitary to secrete ACTH, which stimulates the adrenal glands to secrete cortisol, which then provides negative feedback to both the hypothalamus and pituitary (Figure 7-2). Cortisol is essential for life owing to its many functions, including maintenance of glucose production from protein, facilitation of fat metabolism, augmentation of vascular tone, modulation of central nervous system function, and modulation of the immune system.2

161

FIGURE 7-2 Schematic of the hypothalamic-pituitary-adrenal axis. Regulatory feedback relationships are designated with arrows. (From Mulholland MW, Lillemoe KD,iDoherty GM, Maier RV, SimeoneiDM,tUpchurch GR, eds. Greenfield’s Surgery: Scientific Principles & Pract ce. 5th ed. Philadelphia, PA: Lipp ncot Williams & Wilkins; 2011.)

Are serum cortisol In healthy adults, secretion of cortisol is pulsatile and highest in the early morning.1 levels constant throughout the day? What conditions Stimulants of CRH secretion include stress (eg, trauma, surgery, infection), psychiatric disturbance (eg, depression, normally anxiety), sleep-wake transition, and low serum cortisol.2 stimulate the hypothalamus to secrete corticotropin-releasing hormone? What hormones The adrenal cortex secretes glucocorticoids, mineralocorticoids, and androgens, while the adrenal medulla secretes are secreted by the catecholamines.2 adrenal gland? How common is In the industrialized world, the incidence of adrenal insufficiency is rising; it is currently associated with a prevalence adrenal of up to 40 per 100,000 persons in the general population.1 insufficiency? What are the Clinical manifestations of chronic adrenal insufficiency reflect the consequences of deficient adrenocortical hormones clinical (cortisol, aldosterone, and androgens) and may include fatigue, weakness, malaise, weight loss, nausea, vomiting, manifestations of hypoglycemia, loss of libido (in women), orthostatic hypotension, loss of axillary or pubic hair (in women), and chronic adrenal generalized hyperpigmentation of the skin and mucous membranes (only in chronic primary adrenal insufficiency).1 insufficiency? What are the Acute adrenal insufficiency (ie, adrenal crisis) is usually triggered by acute illness. Manifestations include abdominal clinical pain, nausea, vomiting, fever, confusion, hypotension (usually shock), and hypoglycemia. Many of the symptoms and manifestations of signs can be mistakenly attributed to the acute illness that triggers the adrenal crisis.1 acute adrenal insufficiency? If adrenal In patients with a clinical condition compatible with adrenal insufficiency, an ACTH stimulation test should be insufficiency is performed to confirm the diagnosis of adrenal insufficiency. Before the ACTH stimulation test is performed, a baseline suspected based plasma ACTH level should be drawn, which may later prove useful.1 on the clinical evaluation, what is the next diagnostic step? What are the steps To perform an ACTH stimulation test, a standard dose of synthetic ACTH (250 ”g IV or IM) is given to the patient, of the ACTH and a total serum cortisol level is drawn 60 minutes later. A baseline cortisol level is not necessary because neither the stimulation test? absolute value nor the percentage change between basal and postinjection cortisol has any impact on the interpretation of the ACTH stimulation test.1,3-5 Does the ACTH The ACTH stimulation test can be done at any time of day.3 stimulation test need to be performed at a particular time of day? How should the Normal adrenal function is established when serum cortisol level is ≄18 ”g/dL after ACTH is administered.6 results of the ACTH stimulation test be interpreted? In what scenario The ACTH stimulation test may yield a false-negative result (ie, serum cortisol concentration rises to ≄18 ”g/dL after might the ACTH ACTH is administered in a patient with adrenal insufficiency) in the setting of central adrenal insufficiency of recent stimulation test onset, as there may be incomplete atrophy of the adrenal glands. In such patients, the ACTH stimulation test should yield a false- be repeated a few weeks later. A false-negative result may also occur in patients treated with exogenous negative result? glucocorticoids, particularly hydrocortisone (prednisone and dexamethasone typically do not interfere with modern cortisol assays). The morning dose of hydrocortisone should be held before performing the ACTH stimulation test. It can be given as soon as the blood samples are drawn.1

162

Does a positive stimulation test distinguish between primary and central adrenal ACTH insufficiency? If the ACTH stimulation test is positive, what is the next step to determine whether adrenal insufficiency is primary or central?

A positive ACTHlstimulationitest (ie, serum cortisol concentration remains <18 ”g/dL after ACTH administration) is the expected resu t in any pat ent with chronic adrenal insufficiency; it does not distinguish between primary and central adrenal insufficiency.1

Plasma ACTH level (which should have been drawn before the ACTH stimulation test) determines whether adrenal insufficiency is ACTH-independent (ACTH level is elevated) or ACTH-dependent (ACTH level is low or normal).

In the setting of When there is cortisol deficiency, pituitary ACTH secretion should increase in an attempt to return serum cortisol adrenal levels to normal. Therefore, an ACTH level within the normal range is “inappropriately normal.” insufficiency, why does a normal plasma ACTH value implicate an ACTH-dependent process? Why is it helpful ACTH-independent adrenal insufficiency indicates intrinsic dysfunction of the adrenal glands (primary), whereas to separate the ACTH-dependent adrenal insufficiency indicates a dysfunctional hypothalamic-pituitary axis (central). causes of adrenal insufficiency into ACTH-independent and ACTH-dependent categories?

163

Primary Adrenal Insufficiency

What is the fundamental mechanism of primary adrenal insufficiency? How much adrenal cortical tissue must be destroyed to result in clinically apparent adrenal insufficiency? In addition to cortisol, patients with primary adrenal insufficiency should be evaluated for what other hormonal deficiency? The causes of primary adrenal insufficiency can be separated into which general subcategories?

Primary adrenal insufficiency occurs when the adrenal glands fail to produce adequate hormones despite Increased ACTH stimulation.

Clinically evident adrenal insufficiency occurs when ≄90% of adrenal cortical tissue has been destroyed.7

Mineralocorticoid deficiency may be present in cases of primary ‹adrenal insufficiency, and should be investigated by measuring simultaneous plasma renin activity and serum aldosterone concentration. Mineralocorticoid deficiency due to primary adrenal insufficiency should result in low serum aldosterone levels with elevated plasma renin activity. In such patients, mineralocorticoid replacement therapy will prevent sodium loss, intravascular volume depletion, and hyperkalemia; it is given in the form of fludrocortisone (9-α-fluorohydrocortisone), and the dose is titrated according to blood pressure, serum sodium and potassium concentrations, and plasma renin activity.1,2

The causes of primary adrenal insufficiency can be separated into the following subcategories: autoimmune, infectious, hemorrhagic, infiltrative, and other.

164

Autoimmune Causes of Primary Adrenal Insufficiency

What are the autoimmune causes of primary adrenal insufficiency?

This condition is the most common cause of primary adrenal insufficiency in the industrialized world. Autoimmune conditions often coexist within individuals and families.

How common is autoimmune adrenalitis?

What are the characteristics of polyglandular autoimmune syndrome type 1? What are the characteristics of polyglandular autoimmune syndrome type 2?

Isolated autoimmune adrenalitis.1

Polyglandular autoimmune syndrome type 1 (PAS-1) and polyglandular autoimmune syndrome type 2 (PAS-2).

Autoimmune adrenalitis accounts for up to 90% of cases of primary adrenal insufficiency in the industrialized world. Serum autoantibodies to the adrenal cortex or 21-hydroxylase are present in the vast majority of cases. The finding of small adrenal glands on imaging can be a clue to the diagnosis.1,8,9 PAS-1 is a rare autosomal recessive disorder that disproportionately affects certain populations, such as patients from Sardinia and Finland. Main features include chronic mucocutaneous candidiasis, autoimmune hypoparathyroidism, and autoimmune adrenalitis. Confirmatory testing includes measurement of serum antibodies and gene mutation analysis.1 PAS-2 is more common than PAS-1. It occurs more frequently in women than men and often presents in the fourth decade of life. Main features include autoimmune adrenalitis, autoimmune thyroid disease, and type 1 diabetes mellitus.1

165

Infectious Causes of Primary Adrenal Insufficiency

What are the infectious causes of primary adrenal insufficiency?

A 58-year-old man from Mexico presents with chronic abdominal pain, weight loss, and night sweats, and is found to have calcified and atrophic adrenal glands on cross- sectional imaging. Consider these infections in immunocompromised hosts or anyone with exposure to endemic regions. Associated with a low CD4 cell count.

What percentage of cases of primary adrenal insufficiency are related to tuberculosis?

Which disseminated fungal infections are associated with the development of adrenal insufficiency?

What are the major mechanisms of adrenal insufficiency related to HIV/AIDS?

Tuberculosis.

Disseminated fungal infection.

Human immunodeficiency virus and acquired immunodeficiency syndrome (HIV/AIDS).

When Thomas Addison first described adrenal insufficiency, most cases were related to tuberculosis. It remains a common cause of adrenal insufficiency in the developing world, accounting for up to one-third of cases. In the industrialized world, it accounts for up to 15% of cases. Infection with Mycobacterium tuberculosis involves the adrenal gland via hematogenous spread. Imaging findings consistent with recently acquired disease (<2 years) include bilateral adrenal enlargement, whereas the dominant findings in patients with infection of longer duration are calcification and atrophy.1,10 Virtually any disseminated fungal infection can involve the adrenal gland and lead to primary adrenal insufficiency, but the most common include histoplasmosis, paracoccidioidomycosis (South American blastomycosis), cryptococcosis, blastomycosis, and coccidioidomycosis. In patients with adrenal involvement of disseminated fungal infection, imaging often reveals adrenal gland enlargement; image– guided biopsy can be performed to confirm the diagnosis.10 Adrenal insufficiency occurs in up to one-fifth of patients admitted to the hospital with AIDS, and is related to a variety of mechanisms including infiltration of the glands with opportunistic infection (eg, cytomegalovirus) or HIV-associated malignancy (eg, Kaposi sarcoma, lymphoma), and as a side effect of medications used to treat HIV/AIDS (eg, ketoconazole, fluconazole, rifampin).10

166

Hemorrhagic Causes of Primary Adrenal Insufficiency

Why is the adrenal The adrenal gland is intrinsically susceptible to hemorrhage because of its unique vascular anatomy. Three gland vulnerable to suprarenal arteries supply the gland with a high volume of blood flow, but only 1 vein provides drainage, creating hemorrhage? a “vascular dam.” During periods of hemodynamic stress, such as Increased perfusion pressure, rupture of the capillaries can result in hemorrhage into the gland.7 What clinical clues The combination of hypotension, acute decline in hematocrit, and signs of adrenal insufficiency (eg, hyperkalemia, suggest the hyponatremia, hypovolemia) should prompt consideration of bilateral adrenal hemorrhage. Major risk factors diagnosis of include thromboembolic disease, coagulopathy, and the postoperative state.11 bilateral adrenal hemorrhage? How is the diagnosis Because the clinical manifestations of bilateral adrenal hemorrhage can overlap with concurrent critical illness, of bilateral adrenal diagnosis is often missed or delayed. Diagnosis is confirmed with biochemical evidence of adrenal insufficiency gland hemorrhage and imaging evidence of adrenal hemorrhage (via computed tomography imaging, ultrasonography, or magnetic confirmed? resonance imaging).7 What is the Bilateral adrenal gland hemorrhage is associated with a high mortality rate, largely because of missed or delayed prognosis of diagnosis. In those who survive after treatment with glucocorticoid replacement, the development of chronic bilateral adrenal primary adrenal insufficiency is virtually universal.7 gland hemorrhage?

What are the causes of bilateral adrenal gland hemorrhage?

A 32-year-old man is admitted with Immune thrombocytopenic purpura. epistaxis and petechial skin rash following an upper respiratory tract infection and subsequently develops hypotension, hyperkalemia, and hyponatremia. A vascular complication that can occur in Adrenal vein thrombosis. patients with underlying thrombophilia (eg, antiphospholipid antibody syndrome). Adrenal hemorrhage in the intensive care Critical illness, including the postoperative state, myocardial infarction, congestive heart unit. failure, and sepsis. An 18-year-old woman with headache, Waterhouse-Friderichsen syndrome related to meningococcemia. photophobia, and neck stiffness. A 32-year-old woman is brought to the Trauma. emergency department after a motor vehicle accident, and cross-sectional imaging of the abdomen reveals bilateral adrenal hemorrhage.

Which causes of coagulopathy are most often associated with bilateral adrenal hemorrhage? What is the mechanism of adrenal hemorrhage in the setting of adrenal vein thrombosis? What is the mechanism of bilateral adrenal hemorrhage in the setting of critical illness? What infections are associated with Waterhouse-Friderichsen syndrome?

What types of traumatic injuries are

Thrombocytopenia and the use of anticoagulant medications are the coagulopathies most frequently associated with bilateral adrenal hemorrhage.7,11 The combination of rich arterial blood flow to the adrenal gland and Decreased venous drainage related to adrenal vein thrombosis results in Increased pressure and subsequent capillary rupture with intraglandular hemorrhage.12 It is theorized that surges in ACTH levels during acute illness increase blood flow to the adrenal glands, which overwhelms the drainage capability of the adrenal vein, resulting in capillary rupture and intraglandular hemorrhage.7 Waterhouse-Friderichsen syndrome most commonly occurs in association with sepsis from Neisseria meningitidis. However, infection with other bacteria can also lead to adrenal hemorrhage, including Rickettsia rickettsii, Streptococcus pneumoniae, group A streptococcus, and . Staphylococcus aureus 13 Motor vehicle accidents, falls, and sports injuries are the most common traumatic causes of

167

hemorrhage? location between the liver and spine (Figure 7-3). Bilateral adrenal hemorrhage should be suspected in trauma patients who present with symptoms and signs of adrenal insufficiency. Imaging of the adrenal glands can confirm the diagnosis.14

FIGURE 7-3 Postcontrast CT shows posttraumatic hemorrhage (arrow) into the right adrenal gland. Blunt trauma to the abdomen can compress the right adrenal gland between the liver (L) and the spine (S), resulting in adrenal hemorrhage. This patient also has areas of fracture and hemorrhage (arrowheads) within the liver, as well as a biloma (B). (From Brant W, Helms CA. Fundamentals of Diagnostic Radiology. 3rd ed. Philadelphia, PA: Lippincott Williams & Wilkins; 2007.)

168

Infiltrative Causes of Primary Adrenal Insufficiency

What are the infiltrative causes of primary adrenal insufficiency?

Diffuse lymphadenopathy and elevated serum lactate dehydrogenase. A common histologic feature of sarcoidosis and tuberculosis. Think of this entity in patients with multiple myeloma or chronic inflammatory conditions. Another reason to have bronzed skin.

What are the characteristics of adrenal infiltration related to malignancy?

What are the characteristics of adrenal insufficiency related to sarcoidosis? How common is adrenal insufficiency in patients with systemic amyloidosis with renal involvement? What are the characteristics of adrenal insufficiency related to hemochromatosis?

Lymphoma.

Granulomatous disease.

Amyloidosis.

Hemochromatosis.

Metastatic infiltration is the primary mechanism by which malignancy involves the adrenal gland. Adrenal metastasis occurs most frequently in patients with lung, breast, gastric, and colorectal cancer, melanoma, and lymphoma. Up to one-third of patients with bilateral adrenal metastases develop adrenal insufficiency. Patients with malignancy can also develop adrenal insufficiency as a result of hemorrhagic necrosis, impaired adrenal synthesis due to antineoplastic agents, and central adrenal insufficiency from malignant infiltration of the hypothalamus or pituitary or discontinuation of glucocorticoids used in the antineoplastic treatment protocol.15,16 In patients with sarcoidosis, the adrenal gland can become infiltrated with granulomatous lesions and eventually replaced by dense fibrosis. However, the occurrence of primary adrenal insufficiency is rare in sarcoidosis. Patients generally respond well to glucocorticoid replacement (and mineralocorticoid replacement, if necessary). In addition to its effects on the adrenal glands via granulomatous infiltration, there is an association between sarcoidosis and autoimmune adrenalitis.17 Primary or central adrenal insufficiency occurs in almost one-half of patients with renal amyloidosis. It occurs in both immunoglobulin light chain (AL) amyloidosis and inflammatory (AA) amyloidosis. Symptoms and signs of adrenal insufficiency in these patients may be confused for those of uremia, a common comorbidity in this population.18,19

Iron deposition within the adrenal gland occurs more frequently with secondary hemochromatosis; primary hemochromatosis tends to cause central adrenal insufficiency. In patients with primary adrenal insufficiency related to hemochromatosis, computed tomography imaging shows characteristic hyperdense adrenal glands with normal or reduced size and preserved contours.20,21

169

Other Causes of Primary Adrenal Insufficiency

What are the other causes of primary adrenal insufficiency?

Iatrogenesis (at least 3 answers are correct). An X-linked disease that can be diagnosed by measuring the serum concentration of very long- chain fatty acids.

Bilateral adrenalectomy, radiotherapy, and medication. Adrenoleukodystrophy (ALD).

What type of hormone Lifelong replacement of both glucocorticoids and mineralocorticoids is necessary in all patients who have replacement is necessary undergone bilateral adrenalectomy.22 in patients who have undergone bilateral adrenalectomy? Is external beam radiation Primary adrenal insufficiency related to external beam radiation is relatively rare. In contrast, central adrenal therapy most often insufficiency occurs in up to one-half of patients treated with radiation to the hypothalamic-pituitary region. associated with primary or Median time to occurrence is 5 years.23 central adrenal insufficiency? What are the 2 main Medication-induced primary adrenal insufficiency can occur as a result of (1) inhibition of cortisol mechanisms through biosynthesis (eg, ketoconazole), and (2) acceleration of the metabolism of cortisol (eg, phenytoin).24,25 which medications result in primary adrenal insufficiency? What is ALD is an X-linked genetic condition characterized by impaired peroxisomal ÎČ-oxidation of very long-chain adrenoleukodystrophy? fatty acids, resulting in accumulation within the tissues, including the adrenal cortex. ALD presents with a spectrum of phenotypes, including adrenomyeloneuropathy (AMN), which generally develops in adults in the third and fourth decades of life. The majority of patients with AMN develop adrenal insufficiency.26

170

Central Adrenal Insufficiency

What is the fundamental mechanism of central adrenal insufficiency? What are the clues to the presence of a central process in patients with adrenal insufficiency? Which glands are involved in central adrenal insufficiency?

Is mineralocorticoid deficiency associated with central adrenal insufficiency? What general processes can cause pituitary and/or hypothalamic dysfunction?

What is the most common cause of central adrenal insufficiency?

How long can it take for hypothalamic function to recover once exposure to supraphysiologic doses of glucocorticoids has ceased?

Central adrenal insufficiency occurs as a result of inadequate ACTH stimulation of the adrenal glands.

In patients with adrenal insufficiency, a central process is suggested by preexisting hypothalamic or pituitary disease, history of head trauma, headaches, visual field defects, and focal neurologic findings.27 Central adrenal insufficiency can occur as a result of pituitary gland dysfunction (ie, secondary adrenal insufficiency) or hypothalamic dysfunction (ie, tertiary adrenal insufficiency).

In general, the adrenal glands remain responsive to renin activity in patients with central adrenal insufficiency. However, mineralocorticoid deficiency can develop in cases of longstanding ACTH deficiency.1 Pituitary and/or hypothalamic dysfunction can be caused by medication (eg, glucocorticoids), mass lesion (eg, brain metastasis), traumatic brain injury, subarachnoid hemorrhage, infection/abscess, stroke, external beam radiation, pituitary apoplexy, Sheehan’s syndrome, autoimmune disease (eg, lymphocytic hypophysitis), and infiltrative disease (eg, hemochromatosis).27 Chronic exposure to excessive glucocorticoids, either from an exogenous source (eg, glucocorticoid medication) or from an endogenous source (eg, Cushing’s disease), results in persistent suppression of hypothalamic secretion of CRH. An abrupt decrease in exposure to glucocorticoids, such as when medication is stopped or Cushing’s syndrome is treated, can result in central adrenal insufficiency.1 Recovery of the hypothalamus can take up to 9 months following cessation of exposure to excess glucocorticoids.28

171 Case Summary A 44-year-old man with chronic generalized hyperpigmentation, Gastrointestinal illness and is found to have tachycardia, hypotension, weight loss, and fatigue is admitted to the hospital with an acute hypoglycemia, and hyponatremia.

What is the most likely underlying diagnosis in this patient? Primary adrenal insufficiency.

172

adrenal insufficiency more likely than What is the mechanism of Bonus Questions

Which features of this case make primary central adrenal insufficiency? hyperpigmentation from chronic primary

areas of skin hypopigmentation described in What is the most likely cause of primary adrenal insufficiency? What is the best explanation for the patchy this case? adrenal insufficiency in this case?

Is the patient in this case in adrenal crisis? Which urgent treatment should be provided to the patient in this case?

What is the physiologic replacement dose of glucocorticoids? Does the patient in this case require mineralocorticoid replacement?

The presence of generalized hyperpigmentation (see Figure 7-1A) is the first clue to the diagnosis of chronic primary adrenal insufficiency clinches the diagnosis of primary adrenal insufficiency. in this case. Elevated plasma ACTH in the setting of a compatible clinical syndrome and biochemical evidence of adrenal insufficiency hyperpigmentation. It tends to first occur in areas of the skin under pressure, including elbows, knuckles, palmaricreases, lips, and buccal High levels of ACTH found in patients with primary adrenal insufficiency stimulate the melanocortin-1 receptor n the skin, resulting in mucosa. 1 The patchy areas of hypopigmentation described in this case are most likely manifestations of vitiligo, an autoimmune condition that tends to accompany other autoimmune conditions such as autoimmune adrenalitis.

cause of primary adrenal insufficiency, andtthere are other clues to the diagnosis (eg, vitiligo). There is no evidenceiof a polyglandular Isolated autoimmune adrenalitis is the mos likely cause of primary adrenal insufficiency in this case. In general, it s the most common syndrome based on the information provided. The patient in this case is in adrenal crisis, likely triggered by the underlying inflammatory Gastrointestinal condition. Adrenal crisis is a .1 life-threatening condition characterized by abdominal pain, vomiting, myalgias, arthralgias, severe hypotension, and hypovolemic shock Patients with adrenal crisis should be treated with stress-dose glucocorticoid replacement (typically given as hydrocortisone 50-100 mg intravenously or intramuscularly every 6 hours, depending on age and body surface area), until clinical stability is achieved. At that point, a maintenance dose of glucocorticoid should be started. 1 The physiologic replacement dose (ie, maintenance dose) of glucocorticoids is 10 to 12 mg/m2 per day of hydrocortisone or equivalent.29 The patient in this case might require mineralocorticoid replacement, as mineralocorticoid deficiency occurs in some cases of primary adrenal insufficiency. Testing should be done with simultaneous plasma renin activity and serum aldosterone concentration. 1

173 Key Points

deficient production or actioniof glucocorticoids, with or without Clinical manifestations of chronic adrenal insufficiency include hypoglycemia, loss of libido (in women), orthostatic hypotension, hyperpigmentation of the skin and mucous membranes (only in When adrenal insufficiency is suspected clinically, an ACTH Adrenal insufficiency is a clin cal condition that results from mineralocorticoid or androgen deficiency. fatigue, weakness, malaise, weight loss, nausea, vomiting, loss of axillary or pubic hair (in women), and generalized chronic primary adrenal insufficiency). stimulation test should be performed to confirm the diagnosis. The plasma ACTH level determines whether adrenal insufficiency is ACTH-independent (primary) or ACTH-dependent (central). The causes of primary adrenal insufficiency can be separated into

the following subcategories: autoimmune, infectious, hemorrhagic, dysfunction (secondary) or hypothalamic dysfunctioni(tertiary). withdrawal of glucocorticoid therapy, autoimmune adrenalitis, Acute adrenal insufficiency (ie, adrenal crisis) must be treated with Treatment for chronic adrenal insufficiency includes maintenance-mineralocorticoid replacement in some patients. infiltrative, and other. Central adrenal insufficiency occurs as a result of pitu tary The most common causes of adrenal insufficiency include abrupt and tuberculosis. high-dose glucocorticoids. dose glucocorticoid replacement and may include

174

References 1. Charmandari E, Nicolaides NC, Chrousos GP. Adrenal insufficiency. Lancet. 2014;383(9935):2152-2167.

  1. Berne RML, Levy MN. Physiology. 4th ed. St. Louis, MO: Mosby, Inc.; 1998.
  2. Dorin RI, Qualls CR, Crapo LM. Diagnosis of adrenal insufficiency. Ann Intern Med. 2003;139(3):194-204.

  3. Oelkers W. Adrenal insufficiency. N Engl J Med. 1996;335(16):1206-1212.

  4. after high-dose synthetic ACTH-1-24 injection in the diagnosis of adrenal insufficiency. Zueger T, Jordi M, Laimer M, Stettler C. Utility of 30 and 60 minute cortisol samples Swiss Med Wkly. 2014;144:w13987.

  5. Longo DL, Fauci AS, Kasper DL, Hauser SL, Jameson JL, Loscalzo J, eds. Harrison’s Principles of Internal Medicine. 18th ed. New York, NY: McGraw-Hill; 2012.

  6. Kovacs KA, Lam YM, Pater JL. Bilateral massive adrenal hemorrhage. Assessment of putative risk factors by the case-control method. Medicine. 2001;80(1):45-53.

  7. Kasperlik-Zaluska AA, Migdalska B, Czarnocka B, Drac-Kaniewska J, Niegowska E, Czech W. Association of Addison’s disease with autoimmune disorders—a long-term

observation of 180 patients. Postgrad Med J. 1991;67(793):984-987. 9. Zelissen PM, Bast EJ, Croughs RJ. Associated autoimmunity in Addison’s disease. J Autoimmun. 1995;8(1):121-130.

  1. Upadhyay J, Sudhindra P, Abraham G, Trivedi N. Tuberculosis of the adrenal gland: a case report and review of the literature of infections of the adrenal gland. Int J

Endocrinol. 2014;2014:876037. 11. Rao RH, Vagnucci AH, Amico JA. Bilateral massive adrenal hemorrhage: early recognition and treatment. Ann Intern Med. 1989;110(3):227-235.

  1. Presotto F, Fornasini F, Betterle C, Federspil G, Rossato M. Acute adrenal failure as the heralding symptom of primary antiphospholipid syndrome: report of a case and

review of the literature. Eur J Endocrinol. 2005;153(4):507-514. 13. Guarner J, Paddock CD, Bartlett J, Zaki SR. Adrenal gland hemorrhage in patients with fatal bacterial infections. Mod Pathol. 2008;21(9):1113-1120.

  1. Sinelnikov AO, Abujudeh HH, Chan D, Novelline RA. CT manifestations of adrenal trauma: experience with 73 cases. Emerg Radiol. 2007;13(6):313-318.

  2. Carvalho F,;Louro F, Zakout R. Adrenal insufficiency in metastatic lung cancer. World J Oncol. 2015 6(3):375-377.

  3. Yeung SCJ, EscalanteiCP, Gagel RF, eds..Medical Care of Cancer Patients. Shelton, CT: People’s Medical Publ shing House; 2009
  4. Porter N, Beynon HL, Randeva HS. Endocrine and reproductive manifestations of sarcoidosis. QJM. 2003;96(8):553-561. . Arik N, Tasdemir I, Karaaslan Y, Yasavul U, Turgan C, Caglar S. Subclinical

18adrenocortical insufficiency in renal amyloidosis. Nephron. 1990;56(3):246-248. 19. Danby P, HarrisiKP, Williams B, Feehally J, Walls J. Adrenal dysfunction in patients 20. Doppman JL, Gill Jr JR, Nienhuis AW, Earll JM, Long Jr JA. CT findings in Addison’s with renal amylo d. Q J Med. 1990;76(281):‹915-922. disease. J Comput Assist Tomogr. 1982;6(4):757-761.

  1. Kannan CR. The Adrenal Gland. New York, NY: Plenum Publishing Corporation; 1988.
  2. Loriaux L. The diagnosis and differential diagnosis of Cushing’s syndrome. N Engl J

175

Med. 2017:376;1451-1459. 23. Kufe DW, Pollock RE, Weichselbaum RR, et al, eds. Cancer Medicine. 6th ed. Hamilton (ON): BC Decker; 2003.

  1. Elias AN, Gwinup G. Effects of some clinically encountered drugs on steroid synthesis and degradation. Metabolism. 1980;29(6):582-595.

  2. Sonino N. The use of ketoconazole as an inhibitor of steroid production. N Engl J Med. 1987;317(13):812-818. . Engelen M, Kemp S, de Visser M, et al. X-linked adrenoleukodystrophy (X-AL

26clinical presentation and;guidelines for diagnosis, follow-up and management. D): 27. Persani L. Clinical review: central hypothyroidism: pathogenic, diagnostic, and Orphanet J Rare Dis. 2012 7:51. therapeutic challenges. J Clin Endocrinol Metab. 2012;97(9):3068-3078.

  1. Byyny RL. Withdrawal from glucocorticoid therapy. N Engl J Med. 1976;295(1):30-32.
  2. Gupta P, Bhatia V. Corticosteroid physiology and principles of therapy. Indian J Pediatr. 2008;75(10):1039-1044.

176

CHAPTER 8

177

Cushing’s Syndrome

178 Case: A 43-year-old man with delusions department for evaluation-of confusion. The patientthas complained to months. Over the same period, he has had progressive weakness in his A previously healthy 43 year-old man is brought o the emergency his wife of weight gain and frequent urination for the past several shoulders and legs, recently requiring help to rise from a seated position. The patient’s wife became more concerned when he told her

he is a secret agent of the Federal Bureau of Investigation. The patient Heart rate is 110 beats per minute and blood pressure is has never smoked and does not drink alcohol or use illicit substances. 188/104 mm Hg. There are scattered ecchymoses, abdominal striae (Figure 8-1), and hyperpigmentation of the knuckles, palmar creases, and elbows. Proximal muscle weakness and atrophy are present.

FIGURE 8-1

Urine free cortisol is measured at 645 ”g/dayt(reference range/dL. 1000 pg/mL (reference range 10-60 pg/mL). Simultaneous sampling of reveals a central-to-peripheral ACTH concentration ratio of 0.6. Cross-the left main bronchus (arrow, Figure 8-2). Serum glucose is 525 mg/dL and serum po assium is 2.1 mg <50 ”g/day). Plasma adrenocorticotropic hormone (ACTH) is the inferior petrosal sinus and peripheral blood is performed and sectional imaging of the chest reveals an endobronchial nodule within

FIGURE 8-2 (Courtesy of Cristina Fuss, MD.)

179

What is the most likely diagnosis in this patient?

180

In the setting of cortisol excess, why does a normal plasma ACTH value imply an ACTH-dependent process? What is the relative prevalence of ACTH-dependent and ACTH-independent causes of Cushing’s syndrome?

When there is cortisol excess, negative feedback to the hypothalamus and pituitary should decrease ACTH secretion in an attempt to return serum cortisol levels to normal. Therefore, an ACTH level within the normal range is “inappropriately normal.” ACTH-dependent causes of Cushing’s syndrome represent 80% of cases, whereas ACTH- independent causes make up the remaining 20%.6

181

ACTH-Dependent Cushing’s Syndrome

What is the fundamental mechanism of hypercortisolism in patients with ACTH-dependent Cushing’s syndrome? In patients with ACTH-dependent Cushing’s syndrome, what is the next diagnostic step in evaluation?

ACTH-dependent Cushing’s syndrome occurs as a result of excess ACTH stimulation of the adrenal glands that then respond by producing excess cortisol.

Patients with ACTH-dependent Cushing’s syndrome should undergo inferior petrosal sinus sampling to determine if the excess ATCH is eutopic (from the pituitary gland) or ectopic (from elsewhere). Following direct stimulation of the pituitary gland with CRH, ACTH plasma levels from the inferior petrosal sinus and periphery (eg, antecubital vein) are simultaneously measured. Eutopic ACTH secretion is associated with a central-to- peripheral ACTH ratio ≄3. Ectopic ACTH secretion is associated with a central-to-peripheral ACTH ratio <3.3

182

Cushing’s Syndrome Caused by Eutopic ACTH Production

What are the causes of eutopic ACTH production?

When caused by this underlying condition, Cushing’s syndrome is known as “Cushing’s disease.” The pituitary is overstimulated.

What are the characteristics of ACTH-secreting pituitary adenomas?

Pituitary adenoma.

Excess CRH.

Pituitary adenoma is the most common cause of ACTH-dependent Cushing’s syndrome, about 7 times more common than ectopic sources of ACTH. It is more common in women by a ratio of 4:1, with a peak incidence in the third and fourth decades of life. About half of ACTH-secreting pituitary adenomas are visible on magnetic resonance imaging (MRI) of the brain (Figure 8-4

FIGURE 8-4 Pituitary adenoma. T1-weighted coronal MRI after gadolinium enhancement demonstrates a discrete

What are the sources of excess corticotropin- releasing hormone? What is the most common cause of pseudo-Cushing’s syndrome?

focus of hypointensity (arrow) involving the right side of the pituitary gland, most consistent with pituitary adenoma. (Courtesy of Dr S. Chan.)

). Transsphenoidal adenectomy is the initial treatment of choice for Cushing’s disease. If successful, the plasma cortisol level on the morning after transsphenoidal adenectomy will be zero. Glucocorticoid replacement is necessary until the hypothalamic-pituitary-adrenal axis regains function, which typically takes a yearor longer. Up to one-third of patients will eventually experience a recurrence.3,6 Excess CRH can be caused by pseudo-Cushing’s syndrome or ectopic CRH production from a tumor (rare).

Chronic alcohol use is the most common cause of pseudo-Cushing’s syndrome. Investigation into the possibility of alcohol-induced pseudo-Cushing’s syndrome begins with a period of abstinence from alcohol, followed by clinical monitoring.3

183

Cushing’s Syndrome Caused by Ectopic ACTH Production

What electrolyte disturbance is associated with Cushing’s syndrome caused by ectopic ACTH production?

Hypokalemia is present in the majority of patients with Cushing’s syndrome related to ectopic ACTH production but is only rarely seen in patients with Cushing’s disease. This observation has several explanations. First, compared with those with Cushing’s disease, patients with ectopic ACTH production generally have higher circulating levels of cortisol, which has activity at the mineralocorticoid receptor (which, when activated, promotes renal potassium excretion). Second, the activity of 11ÎČ-hydroxysteroid dehydrogenase type 2, which is essential in preventing the mineralocorticoid activity of cortisol, is Decreased in patients with ectopic ACTH production.7

What are the causes of ectopic ACTH production?

Smoking is the most Small cell lung cancer. important risk factor for the development of this malignancy. This neuroendocrine Carcinoid tumor. tumor, when located in the Gastrointestinal tract, can be associated with flushing and diarrhea. A pancreatic mass is Pancreatic islet cell tumor (ie, pancreatic neuroendocrine tumor). identified on cross- sectional imaging in a patient with Cushing’s syndrome. A type of thyroid Medullary thyroid carcinoma. cancer that originates from the parafollicular cells (C cells). A 32-year-old man Pheochromocytoma. presents with episodes of headache, chest pressure, tachycardia, and hypertension.

What are the characteristics of Cushing’s syndrome related to small cell lung cancer?

What are the most common locations of carcinoid tumors associated with ectopic production of ACTH? What are the characteristics of pancreatic islet cell tumors that secrete ACTH?

Patients with ectopic ACTH production from small cell lung cancer are more likely to present with weight loss, hypokalemia, abnormal glucose tolerance, and edema rather than the more classic manifestations of Cushing’s syndrome. In addition to ectopic ACTH production, small cell lung cancer can cause other paraneoplastic endocrine syndromes, including the syndrome of inappropriate antidiuretic hormone (SIADH), and hypercalcemia via parathyroid hormone–related peptide (PTHrP). Treatment of Cushing’s syndrome related to small cell lung cancer includes radical excision of the tumor, chemotherapy, and pharmacologic cortisol inhibition (eg, ketoconazole). Prognosis is poor.8 Carcinoid tumors of the lung, thymus, and pancreas are most often associated with ectopic ACTH production. Thymic carcinoid tumors in particular are associated with poor prognosis.7

Pancreatic islet cell tumors account for up to 3% of pancreatic tumors. These tumors are capable of secreting various hormones, including insulin, gastrin, glucagon, and ACTH. When the tumor is limited to the pancreas, secreted ACTH enters the enterohepatic circulation and is rapidly metabolized by the liver, preventing the clinical syndrome from developing. By the time Cushing’s syndrome has become apparent, most ACTH-secreting pancreatic islet cell tumors are advanced, with hepatic metastases, and are associated with poor prognosis.9

184

Which genetic Medullary thyroid carcinoma and pheochromocytoma can occur together in multiple endocrine neoplasia types syndrome is 2a and 2b but are most often sporadic. These entities are relatively rare sources of ectopic ACTH production.6 associated with the development of both medullary thyroid carcinoma and pheochromocytoma? In patients with Patients with ectopic sources of ACTH production should undergo an imaging study guided by the clinical Cushing’s syndrome presentation (eg, a patient with a history and examination compatible with pheochromocytoma should undergo caused by ectopic imaging of the adrenal glands). When there are no clinical clues to a possible tumor site, patients should first ACTH production, undergo computed tomography imaging or MRI of the chest; a tumor will be found in the majority of patients. If what is the next a tumor is not identified in the chest, then MRI of the abdomen and pelvis should be performed next.3 diagnostic step? If the source of ectopic If a tumor cannot be identified in patients with ectopic ACTH production, there are 2 options for management: ACTH production pharmacologic blockade of cortisol synthesis or bilateral adrenalectomy.3 cannot be identified after imaging the chest, abdomen, and pelvis, what treatment options are available?

185

ACTH-Independent Cushing’s Syndrome

What is the fundamental mechanism of hypercortisolism in patients with ACTH-independent Cushing’s syndrome?

ACTH-independent Cushing’s syndrome occurs as a result of excess cortisol production by the adrenal glands, independent of ACTH stimulation.

What are the ACTH-independent causes of Cushing’s syndrome?

Cross-sectional imaging of Adrenal tumor and adrenal hyperplasia. the adrenal ‹glands can be helpful in distinguishing ‹ these 2 causes of ACTH-independent Cushing’s syndrome.

What are the characteristics of adrenal tumors that cause Cushing’s syndrome?

What are the 2 general types of ACTH-independent adrenal hyperplasia?

Adrenal tumors capable of causing Cushing’s syndrome via secretion of excess cortisol include both adenomas (benign) and carcinomas (malignant). Benign tumors tend to be smaller in size (<5 cm) and secrete only 1 hormone (eg, cortisol). These tumors are treated with laparoscopic adrenalectomy with a high rate of success. Malignant tumors tend to be larger in size (>5 cm) and secrete more than 1 hormone (eg, cortisol and androgen). Surgical removal of all detectable tissue, including metastases, should be pursued.3 Cushing’s syndrome can occur as a result of micronodular adrenal hyperplasia or bilateral macronodular adrenal hyperplasia. These entities typically affect both adrenal glands. Bilateral adrenalectomy is curative. All patients who undergo bilateral adrenalectomy must be treated with lifelong glucocorticoid and mineralocorticoid replacement.3

186 Case Summary A 43-year-old man presents with weight gain, polyuria, and

confusion, and is found to have ecchymoses, abdominal striae, and hyperpigmentation on examination and an endobronchial nodule

within the left main bronchus on cross-sectional imaging of the chest. What is the most likely diagnosis in this patient? Cushing’s syndrome related to ectopic ACTH production.

187

Bonus Questions

Which clinical features in Features of Cushing’sisyndrome in this case include central obesity, psychosis, ecchymoses, abdominal striae (see Figure 8-1), polyuria (from diagnosis of Cushing’s this case suggest the hyperglycemia), arter al hypertension, proximal myopathy, hyperpigmentation, and hypokalemia. syndrome? the hyperpigmentation in f resulting in hyperpigmentation. It tends to first occur in areas of the skin under pressure, including elbows, knuckles, palmar creases, lips, and buccal skin, What is the significance o Hyperpigmentation is associated only with ACTH-dependent Cushing’s syndrome. High levels of ACTH stimulate the melanocortin-1 receptor in the this case? mucosa. What is the most l The patient in this case has ACTH-dependent Cushing’s syndrome based on the elevated plasma ACTH level. Inferior petrosal sinus sampling demonstrat 2 cause of Cushing’sikely a central-to-peripheral,ACTH ratio <3, implying an ectopic source of ACTH production. In such patients, cross-sectional imaging of the chest often reveals es syndrome in this case? the source. In this case the endobronchial nodule within the left main bronchus (see Figure 8-2, arrow) is most likely a bronchial carcinoid tumor. Bronchoscopy with biopsy would confirm the diagnosis. What other type of lungh? Cushing’s.syndrome.can also occur in patients with small cell lung cancer. However, these patients are less likely to present with the classic manifestations of choiceifor Cushing’snt of or the tumor cannot be located, other treatment options include pharmacologic cortisol synthesis blockade (eg, ketoconazole), and bilateral adrenalectomy.3 cancer is associated wit Cushing’s syndrome Bronchial carcinoid tumors tend to follow a more chronic and indolent course, which allows for full development of Cushing’s ectopic ACTH production syndrome 8 What s the treatme Cushing’s syndrome related to ectopic ACTH production should be treated with surgical removal of the tumor if it can be located. If surgery is not possible syndrome related to ectopic ACTH production?

188 Key Points

Cushing’s syndrome is a clinical condition that results from cortisol excess. Clinical manifestations of Cushing’s syndrome include central obesity, moon facies, buffalo hump, thin skin, bruising, abdomina

striae, hyperpigmentation, hirsutism, oligomenorrhea, psychosis, l When Cushing’s syndrome is.suspected clinically, it should be proximal myopathy, arterial hypertension, hyperglycemia, hypokalemia, and osteopenia confirmed with the UFC test. If the UFC test is negative, it may be a false-negative result or the Plasma ACTH level determines whether Cushing’s syndrome is cases). patient may be receiving exogenous glucocorticoids. ACTH-dependent (80% of cases) or ACTH-independent (20% of ACTH-dependent Cushing’s syndrome occurs as a result of eutopic (pituitary) or ectopic ACTH excess. Inferior petrosal sinus sampling is used to distinguish eutopic and ectopic sources of ACTH production.

ACTH-independent Cushing’s syndrome occurs as a result of cortisol excess from the adrenal gland(s). Treatment for Cushing’s syndrome may involve surgical and

pharmacologic modalities, depending on the underlying cause.

189

References 1. Berne RML, Levy MN. Physiology. 4th ed. St. Louis, MO: Mosby, Inc.; 1998. 2. Charmandari E, Nicolaides NC, Chrousos GP. Adrenal insufficiency. Lancet. 2014;383(9935):2152-2167.

  1. Loriaux DL. Diagnosis and differential diagnosis of Cushing’s syndrome. N Engl J Med. 2017;376(15):1451-1459.

  2. Loriaux DL. Diagnosis and differential diagnosis of Cushing’s syndrome. N Engl J Med. 2017;377(2):e3. Nieman LK, Biller BM, Findling JW, et al. Treatment of Cushing’s syndrome: an

  3. Endocrine Society Clinical Practice Guideline. J Clin Endocrinol Metab. 2015;100(8):2807- 2831.

  4. Lacroix A, Feelders RA, Stratakis CA, Nieman LK. Cushing’s syndrome. Lancet. 2015;386(9996):913-927.

  5. Salgado LR, Fragoso MC, Knoepfelmacher M, et al. Ectopic ACTH syndrome: our experience with 25 cases. Eur J Endocrinol. 2006;155(5):725-733.

  6. Gandhi L, Johnson BE. Paraneoplastic syndromes associated with small cell lung cancer. J Natl Compr Canc Netw. 2006;4(6):631-638. Byun J, Kim SH, Jeong HS, Rhee Y, Lee WJ, Kang CM. ACTH-producing

  7. neuroendocrine tumor of the pancreas: a case report and literature review. Ann Hepatobiliary Pancreat Surg. 2017;21(1):61-65.

190

CHAPTER 9

191

Hypercalcemia

192 Case: A 71-year-old man with confusion anA 71-year-old black man with a history of coronary artery disease frequent urination. Medications include aspirin, simvastatin,ain and veHeart rate isr110 beats per minute. The patient appears ill. Jugulary. 110 mEq/L, serum bicarbonate 22 mEq/L, serum creatinine 2.3 mg/dL, serum albumin 2.5 g/dL, and total serum protein 8.4 g/dL. Serum demonstrates a peripheral density with smooth borders (arrow, Figure d hypertension is brought to the emergency department for evaluation of confusion. Symptoms of malaise and fatigue began several weeks ago. More recently, he developed abdominal p hydrochlorothiazide, and metoprolol. 2 nous pressu e is 4 cm H O. Mucous membranes and axillae are dr Hematocrit is 28%, serum sodium 134 mEq/L, serum chloride total serum calcium 14.6 mg/dL (reference range 8.6-10.4 mg/dL), parathyroid hormone (PTH) is undetectable. Chest radiograph 9-1).

FIGURE 9-1

What is the most likely cause of hypercalcemia in this patient?

What is the normal Normal total serum calcium in adults is 8.6 to 10.4 mg/dL. Severity of hypercalcemia is variably defined, but the range for total following thresholds provide a rule of thumb: total serum calcium 10.5 to 11.9 mg/dL is mild, total calcium 12.0 to serum calcium 13.9 mg/dL is moderate, and total serum calcium >14.0 mg/dL is severe.1,2 concentration? How is calcium The vast majority of calcium (99%) is stored within bone, while approximately 1% is found within intracellular distributed in the fluid, and about 0.1%, in extracellular fluid. Total body calcium is determined by Gastrointestinal absorption and body? renal excretion.3 How is calcium Under normal conditions, total serum calcium is divided between 3 forms: roughly 45% of total serum calcium is distributed within bound to plasma proteins (mostly albumin), 10% is bound to anions (eg, phosphate and citrate), and 45% is ionized blood? (unbound). Only ionized calcium is biologically active.1 What conditions Conditions that may change ionized calcium without changing total calcium concentration include alkalemia may result in a (decreases ionized fraction), acidemia (increases ionized fraction), and the presence of calcium chelators (decreases change in ionizedtal ionized fraction). When these conditions are suspected, ionized calcium should be measured directly.1,2 calcium but not to serum calcium concentration? What conditions Conditions that may change total calcium without changing ionized calcium concentration include may result in a hypoalbuminemia (decreases total calcium), hyperalbuminemia (increases total calcium), and multiple myeloma change in total (increases total calcium).1,2 serum calcium but not ionized calcium concentration?

193

measurement of total serum calcium concentration be adjusted in the setting of hypoalbuminemia? What processes are responsible for regulating serum calcium concentration?

(g/dL)]). When significant changes in serum protein levels are suspected, ionized calcium should be measured directly.1,2

Serum ionized calcium concentration is tightly regulated by the actions of PTH and activated vitamin D (ie, 1,25(OH)2D, or 1,25-dihydroxyvitamin D). Hypocalcemia stimulates the secretion of PTH from the parathyroid glands and the production of activated vitamin D within the proximal tubule cells of the kidney. PTH acts directly at 2 sites in the body to increase serum calcium: the bone (by stimulating osteoclasts) and the kidney (to increase proximal tubular reabsorption of calcium). By stimulating production of 1,25(OH)2D, PTH acts indirectly in the GI tract to increase calcium absorption. Through a negative-feedback mechanism, PTH secretion from the parathyroid glands is turned off when calcium levels are normal or elevated (Figure 9-2).3

FIGURE 9-2 Summary of the actions of PTH on bones, kidneys, and intestines. Decreased serum calcium concentration

Why is it important to maintain normal serum calcium concentration?

What are the symptoms of hypercalcemia?

What are the electrocardiographic manifestations of hypercalcemia? What is the first step in determining the etiology of hypercalcemia?

In the setting of hypercalcemia, why does a normal serum PTH value implicate a PTH- dependent process?

is the primary stimulus for PTH secretion by the parathyroid glands. PTH raises serum calcium levels through its effects on bones, kidneys, and intestines. (From Golan DE, Armstrong EJ, Armstrong AW.iPrinciples of Pharmacology: The Pathophysiologic Basis of Drug Therapy. 4th ed. Philadelph a, PA: Wolters Kluwer Health; 2017.)

Hypocalcemia increases the excitability of nerve and muscle cells, which can lead ‹to tetany; hypercalcemia decreases neuromuscular excitability, which can lead to cardiac dysrhythmias, lethargy, disorientation, and death. Clinical manifestations associated with hypercalcemia increase in severity based on not only the degree of elevation in serum calcium, but also the rate at which it develops. Moreover, older patients with preexisting cognitive dysfunction may experience neurologic complications with milder degrees of hypercalcemia compared with younger, previously healthy patients.1-3 Mild and moderate hypercalcemia are commonly asymptomatic, depending on the rate of development and other patient-specific factors. Hypercalcemia typically affects the neuropsychiatric, Gastrointestinal, and renal systems. Neuropsychiatric symptoms may include anxiety, mood changes, and Decreased cognitive function. Gastrointestinal symptoms may include anorexia, constipation, abdominal pain, nausea, and vomiting. Renal symptoms may include polyuria, polydipsia, and symptoms of nephrolithiasis.1,2,4 Hypercalcemia typically causes shortening of the QT interval. Severe cases can mimic the changes associated with ST-elevation myocardial infarction. Cardiac dysrhythmias can occur, including highly unstable rhythms such as ventricular fibrillation.4 Serum PTH level determines whether hypercalcemia is PTH-dependent (PTH level is elevated or normal) or PTH- independent (PTH level is low).

When serum calcium is elevated, negative feedback to the parathyroid glands should turn off PTH secretion in an attempt to return serum calcium levels to normal. Therefore, a PTH level within the normal range is “inappropriately normal.”

194

195

PTH-Dependent Hypercalcemia

What is the fundamental mechanism of PTH- dependent hypercalcemia?

PTH-dependent hypercalcemia occurs as a result of excess PTH secretion from the parathyroid glands despite elevated serum calcium levels.

What are the causes of PTH-dependent hypercalcemia?

The single most common cause of hypercalcemia. Associated with end-stage renal disease. This genetic syndrome is associated with low urine calcium concentration.

Primary hyperparathyroidism.5 Tertiary hyperparathyroidism. Familial hypocalciuric hypercalcemia (FHH).

What are the characteristics of primary hyperparathyroidism?

What clinical features raise suspicion that primary hyperparathyroidism may be part of an underlying multiple endocrine neoplasia syndrome? What clinical features raise suspicion that primary hyperparathyroidism is caused by parathyroid carcinoma? Is secondary hyperparathyroidism associated with hypercalcemia?

What is tertiary hyperparathyroidism?

What is the underlying mechanism of hypercalcemia in familial hypocalciuric hypercalcemia?

Primary hyperparathyroidism can occur as a result of parathyroid adenoma, ‹hyperplasia, or carcinoma (rare). It can be sporadic or part of the multiple endocrine neoplasia (MEN) types 1 and 2a syndromes. The majority of cases occur in women, and incidence peaks in the seventh decade of life (incidence is similar between men and women before 45 years of age). In the industrialized world, most patients are asymptomatic and the disease is discovered with routine laboratory evaluation, although worldwide, a significant proportion of patients are symptomatic. Treatment is surgical. Asymptomatic patients can be monitored over time.5 The possibility that primary hyperparathyroidism is part of an underlying MEN syndrome increases with the presence of skin lesions associated with MEN (eg, angiofibroma) and when patients are young (<30 years), have a family history of hypercalcemia, and have a personal or family history of neuroendocrine tumors.5

The possibility that primary hyperparathyroidism is caused by parathyroid carcinoma increases when there is a palpable neck mass, severe hypercalcemia (>14 mg/dL), and markedly elevated PTH levels (greater than 3-10 times the upper limit of normal). Parathyroid carcinoma is a rare cause of hypercalcemia (<1% of cases).5

Secondary hyperparathyroidism is not associated with hypercalcemia. In fact, it is usually associated with normal or low serum calcium levels. Secondary hyperparathyroidism occurs in patients with compromised renal function. In these patients, PTH secretion is stimulated in response to phosphate retention and Decreased activated vitamin D levels. Serum calcium levels may be preserved initially, but as renal dysfunction progresses, the compensatory response begins to fail.6 Tertiary hyperparathyroidism can occur after chronic secondary hyperparathyroidism is corrected. Long-standing secondary hyperparathyroidism (usually from renal failure) results in hypertrophy of the parathyroid glands, which can continue to secrete PTH in excess for prolonged periods after the underlying cause of secondary hyperparathyroidism has been corrected (eg, hemodialysis is started). In FHH, a mutation in the calcium-sensing receptor expressed in parathyroid tissue causes Decreased sensitivity to serum calcium levels. This results in an increase in the “set point” of serum calcium, leading to inappropriate secretion of PTH despite elevated serum calcium.5

196

PTH-Independent Hypercalcemia

What is the fundamental mechanism of PTH- independent hypercalcemia? The causes of PTH-independent hypercalcemia can be separated into which general subcategories?

PTH-independent hypercalcemia occurs despite normal parathyroid gland function and Decreased (often undetectable) serum PTH levels. The causes of PTH-independent hypercalcemia can be separated into the following subcategories: medication, malignancy, granulomatous disease, Endocrinopathy, and other.

197

PTH-Independent Hypercalcemia Related to Medication

Which medications cause PTH-independent hypercalcemia?

A 40 year-old woman uses over-the-counter medication to treat severe acid reflux disease. Vitamins. A 52-year-old man develops hypercalcemia after starting treatment for hypertension. Hypercalcemia in a patient with bipolar disorder.

What is the milk-alkali syndrome?

What is the safe limit for daily vitamin D intake? What are the dietary sources of preformed vitamin A? What is the mechanism of hypercalcemia related to thiazide diuretic use? What is the problem with relying on symptoms to diagnose hypercalcemia related to lithium use in bipolar patients?

Milk-alkali syndrome.

Vitamins D and A. Thiazide diuretics.

Lithium.

Milk-alkali syndrome is characterized by the development of hypercalcemia with hypercalciuria and renal dysfunction in patients who consume large quantities of calcium (at least 2 g of elemental calcium per day) and absorbable alkali (eg, calcium carbonate). The history may be readily informative, but sometimes the nature of the dietary source may be harder to recognize (eg, the combination of betel nuts with oyster shell powder). In some cases, the patient may intentionally or unintentionally conceal the ingestion.7,8 The dose at which vitamin D becomes toxic is not clearly known, but the recommended upper limit of daily intake is 100 ”g (4000 IU).9 The most common dietary sources of preformed vitamin A include multivitamins, fish liver oil, animal liver, and fortified foods including milk, butter, margarine, and breakfast cereals. Chronic toxicity develops when patients consume large amounts of preformed vitamin A over months to years.10 Thiazide diuretics can increase the reabsorption of calcium in the kidney. Hypercalcemia occurs in up to 8% of patients taking thiazides.1

The symptoms of hypercalcemia can mimic those of the underlying mental health disorder being treated with lithium. Hypercalcemia occurs in approximately 15% of patients taking lithium. The mechanisms of lithium-induced hypercalcemia are direct stimulation of PTH secretion and Increased renal calcium reabsorption. It is reversible with careful discontinuation of the drug.1,11

198

PTH-Independent Hypercalcemia Related to Malignancy

What are the 3 main mechanisms of PTH-independent hypercalcemia related to malignancy?

The most common mechanism of hypercalcemia associated with malignancy, responsible for 80% of cases. Sometimes visible on imaging; this condition is responsible for 20% of cases of hypercalcemia related to malignancy. This is also the mechanism of hypercalcemia related to sarcoidosis.

Which malignancies are most commonly associated with hypercalcemia? What is the mechanism of PTHrP- induced hypercalcemia?

Secretion of parathyroid hormone–related peptide (PTHrP).4

Osteolytic metastases.4

Ectopic 1,25(OH)2D secretion.

Malignancies most frequently associated with hypercalcemia include breast cancer, lung cancer, and multiple ‹myeloma.12 PTHrP is close in structure to PTH and acts on the same receptors, with overlapping effects. Most cases of hypercalcemia related to PTHrP are associated with breast cancer, squamous cell cancer, renal cell carcinoma, bladder cancer, ovarian cancer, non-Hodgkin’s lymphoma, and endometrial cancer.2,4

FIGURE 9-3 Lateral radiograph of the distal femur of a 65-year-old woman with multiple myeloma

Which malignancies are capable of causing hypercalcemia through local osteolysis? Which malignancies are capable of causing hypercalcemia through the production of 1,25(OH)2D?

showing multiple lytic lesions, which produce a characteristic “moth-eaten” appearance. (From Greenspan A. Orthopedic Imaging: A Practical Approach. 5th ed. Philadelphia, PA: Lippincott Williams & Wilkins; 2011.)

Most cases of hypercalcemia related to local osteolysis are associated with breast cancer and multiple myeloma (Figure 9-3). Lymphoma is also capable of causing hypercalcemia through this mechanism.2,4 Most cases of hypercalcemia from ectopic production of 1,25(OH)2D are related to lymphoma and some ovarian germ cell tumors.2,4

199

200

PTH-Independent Hypercalcemia Related to Granulomatous Disease

What is the mechanism of hypercalcemia related to granulomatous disease? What medication can be helpful in treating the hypercalcemia associated with granulomatous disease?

Granulomatous diseases can cause hypercalcemia as a result of ectopic secretion of 1,25(OH)2D. The hypercalcemia of granulomatous disease can be aggravated by a diet rich in vitamin D and excessive sun exposure. Serum levels of 1,25(OH)2D can be measured to support the ‹diagnosis.13 Glucocorticoids can treat the hypercalcemia associated with granulomatous disease by decreasing the production of 1,25(OH)2D through the inhibition of 1-α-hydroxylase and stimulating its metabolism through the activation of 24-hydroxylase.1,14

What are the granulomatous causes of PTH-independent hypercalcemia?

A 52-year-old woman presents with chronic fatigue and dyspnea and is found to have hypercalcemia and bilateral hilar lymphadenopathy (see Figure 21-4). A 39-year-old expatriate of Mexico presents with weight loss, night sweats, and hemoptysis. Saddle nose deformity (see Figure 50-4). A 34-year-old spelunker presents with fever and cough. Abdominal pain and diarrhea. Between lithium and boron.

What are the characteristics of hypercalcemia related to sarcoidosis?

What medication can be added to the antimicrobial treatment for tuberculosis to lower calcium levels? What is the prognosis of hypercalcemia related to granulomatosis with polyangiitis? Where is Histoplasma capsulatum endemic?

Is Crohn’s disease more commonly associated with hypercalcemia or hypocalcemia? Beryllium exposure most frequently occurs in what industries? What other granulomatous conditions can cause hypercalcemia?

Sarcoidosis.

Tuberculosis (TB).

Granulomatosis with polyangiitis (GPA, or Wegener’s granulomatosis). Histoplasmosis. Crohn’s disease. Berylliosis.

Hypercalcemia occurs in around one-fifth of patients with sarcoidosis. It can develop in any patient with sarcoidosis regardless of race, age, or sex. It occurs more frequently in patients with systemic sarcoidosis compared with those with limited disease. Most patients have mild hypercalcemia, but severe life-threatening hypercalcemia can occur.15 In patients with hypercalcemia related to TB, glucocorticoids can be added to the antituberculosis regimen to control calcium levels.13 The hypercalcemia of GPA is responsive to treatment for the underlying condition with glucocorticoids and cyclophosphamide.13 Histoplasma capsulatum is endemic to the Midwestern and South Central regions of the United States, and most cases originate there. Histoplasmosis has also been reported in Southeast Asia (eg, Indonesia, Thailand, Vietnam).16 Although Crohn’s disease can be associated with hypercalcemia, hypocalcemia is more common because of poor calcium intake and vitamin D deficiency related to malabsorption.17 Beryllium is used in automotive electronics, telecommunications, computers, aerospace, and defense equipment. Many workers are unaware of the exposure.18 Other granulomatous conditions associated with hypercalcemia include fungal infections (eg, coccidioidomycosis), leprosy, and foreign-body granulomatous reactions (eg, silicosis).13

201

PTH-Independent Hypercalcemia Related to Endocrinopathy

What are the endocrinologic causes of PTH-independent hypercalcemia?

Associated with a fine tremor. Hypoglycemia, hyponatremia, and hypercalcemia. A disease that does not respond well to ÎČ-blocker monotherapy. A 43-year-old woman goes up in shoe size for the first time in 30 years.

Thyrotoxicosis. Adrenal insufficiency.

Pheochromocytoma.

Acromegaly.

What are the Hypercalcemia occurs in around one-fifth of patients with thyrotoxicosis and is typically mild to moderate in characteristics of severity. The mechanism of hypercalcemia related to thyrotoxicosis is thought to be Increased bone turnover. hypercalcemia related to It is reversible with treatment of thyrotoxicosis.19 thyrotoxicosis? What are the Hypercalcemia is not uncommon in patients with adrenal insufficiency, particularly during acute episodes characteristics of (ie, adrenal crisis). Volume contraction results in hemoconcentration and Increased renal calcium hypercalcemia related to reabsorption. It is reversible with treatment of adrenal insufficiency.20 adrenal insufficiency? What are the mechanisms Hypercalcemia can develop in patients with pheochromocytoma as a result of ectopic secretion of PTHrP, of hypercalcemia related to catecholamine-induced osteoclastic bone resorption, and catecholamine-induced PTH secretion. It is pheochromocytoma? reversible with removal of the tumor.21 What is the most common Hypercalcemia related to acromegaly is most commonly the result of coexisting primary cause of hypercalcemia in hyperparathyroidism. However, when it occurs independently, the mechanism is thought to be patients with acromegaly? overproduction of 1,25(OH)2D. It is typically mild in severity and, in most cases, resolves with treatment of acromegaly. 22 Some endocrinopathies occur in association with multiple endocrine neoplasia syndromes, and hypercalcemia may be driven by primary hyperparathyroidism in those cases.

202

Other Causes of PTH-Independent Hypercalcemia

What are the other causes of PTH-independent hypercalcemia?

A 25-year-old man develops hypercalcemia 2 months after a car accident that has left him bedbound. A patient with a crush injury initially presents with hypocalcemia but subsequently develops hypercalcemia.

What are the characteristics of hypercalcemia related to immobilization? What are the characteristics of hypercalcemia related to rhabdomyolysis?

Immobility.

Rhabdomyolysis.

Hypercalcemia from immobilization typically affects children or young adults during the first 4 to 6 weeks after an inciting event; however, it can happen months later. It is caused by Increased bone resorption. Bisphosphonates are useful in treating this condition. Physical therapy should also be used to improve the underlying condition.23 Calcium dysregulation occurs in patients with rhabdomyolysis complicated by Acute Kidney Injury. Hypocalcemia is the dominant finding in early rhabdomyolysis, during the oliguric phase of kidney injury; it is likely related to calcium deposition in injured tissues. Around one-third of these patients develop hypercalcemia during the diuretic phase of Acute Kidney Injury related to the remobilization of calcium from the soft tissues. Increased 1,25(OH)2D levels may also play a role.24

203 Case Summary A 71-year-old black man with a history of hypertension treated with anemia, kidney injury, hypercalcemia, elevated protein gap, and an a thiazide diuretic presents with confusion and is found to have abnormal chest radiograph.

What is the most likely cause of hypercalcemia in this patient? Multiple myeloma.

204

Bonus Questions

What is multiple Multiple myeloma is alclonal plasma,cell malignant neoplasm with characteristic clinical manifestations. Multiple myeloma occurs more frequently in men and What is the of the Because of the presence of cationic paraproteins, the anion gap is frequently low or negative in patients with multiple myeloma. In this case, the low anion gap of myeloma? blacks, and is almost a ways preceded by monoclonal gammopathy of unknown significance. The median age at onset is 66 years. The most common clinical Figure 9-3 manifestations are fatigue, bone pain anemia, kidney injury, hypercalcemia, and osteolytic skeletal lesions (see ). 25 anion gap in this significance 2 mEq/L is a clue to the diagnosis. 26 case? Which laboratory The diagnosis of multiple myeloma is based on the presence of a monoclonal protein in serum or urine and the presence of ≄10% monoclonal bone marrow tests shou d be plasma cells. Serum protein electrophoresis can be used to identify a monoclonal (M) protein, which is characterized as a narrow spike in the Îł zone of the g ordered tolinvestigate Serum immunofixation can then confirm that the gammopathy is monoclonal rather than polyclonal in nature. A monoclonal protein is identified on serum el. the elevated total protein electrophoresis in 80% of patients with multiple myeloma. Adding serum immunofixation and the serum free light chain assay (or 24-hour urinary protein in this case? protein electrophoresis with immunofixation) will identify almonoclonal protein in nearly all cases. About 2% to 3% of cases are not associated with a detectable monoclonal protein and are referred to as “nonsecretory mu tiple myeloma.” In such cases, the diagnosis is based on the presence of ≄30% monoclonal bone marrow plasma cells, or a biopsy-proven plasmacytoma. What is the The peripheral density in the chest radiograph in this case (see Figure 9-1, arrow) has a “ball-under-the-rug” appearance, which indicates that the lesion arises 25,27

significance of the from outside the parietal pleura of the lung (eg, rib metastasis, nerve sheath tumor, lipoma). In this case, the most likely explanation is a plasmacytoma arising radiographic from the chest wall or rib. 28 abnormality in this case? What is the of the use Although it is not the principal cause of hypercalcemia in this case, thiazide diuretics can contribute to hypercalcemia and should be stopped in patients with hydrochlorothiazide of significance hypercalcemia from other causes. by the patient in this case? What is the corrected Corrected total serum calcium (g/dL) =14.6 g/dL +(0.8 ×[4 – 2.5 g/dL]) =15.8 g/dL. concentration in this serum total calcium case? What treatment(s) Patients with hypercalcemia are often dehydrated as a result of hypercalcemia-mediated nephrogenic diabetes insipidus, and Decreased oral intake re ated to can be used to acutely Gastrointestinalisymptoms. Intravenous crystalloid solutions can be used to expand intravascular volume and decrease reabsorption of calcium in thelproximal Which class ofn be Bisphosphonates are the long-term treatment of choice forihypercalcemia and act by decreasing bone resorption through the inhibition of osteoclast activity. The levels? lower serum calcium tubule. Calciton n can also be used to acutely decrease serum calcium by inhibiting bone resorption and decreasing renal tubular calcium reabsorption. 1 used for long-term management of medications ca calcium-lowering effect of bisphosphonates typically requ res several days. 1 hypercalcemia in this case (not including treatment for the

underlying disease)?

205 Key Points

Serum calcium levels are tightly regulated by the actions of PTH Hypercalcemia decreases neuromuscular excitability with cognitive,function,,anorexia, constipation, abdominal pain, nausea, Clinical manifestations associated with hypercalcemia increase in and the active form of vitamin D. manifestations that include anxiety, mood changes, Decreased vomiting polyuria polydipsia, QT shortening, and cardiac dysrhythmia. severity based on not only the degree of elevation in serum calcium, but also the rate at which it develops.

Serum PTH level determines whether hypercalcemia is PTH- dependent or PTH-independent.

The most common cause of PTH-dependent hypercalcemia (and of The causeslof PTH-independent:hypercalcemia can be separated Intravenous fluids and calcitonin can be used to acutely decrease Hypercalcemia can be managed long-term with bisphosphonates hypercalcemia overall) is primary hyperparathyroidism. into the fol owing subcategories medication, malignancy, granulomatous disease, Endocrinopathy, and other. serum calcium levels. and by treating the underlying condition.

206

References 1. Minisola S, Pepe J, Piemonte S,:Cipriani C. The diagnosis and management of hypercalcaemia. BMJ. 2015;350 h2723. 2. Stewart AF. Clinical practice. Hypercalcemia associated with cancer. N Engl J Med. 2005;352(4):373-379.

  1. Berne RML, Levy MN. Physiology. 4th ed. St. Louis, MO: Mosby, Inc.; 1998.
  2. Mirrakhimov AE. Hypercalcemia of malignancy: an update on pathogenesis and management. N Am J Med Sci. 2015;7(11):483-493.

  3. Marcocci C, Cetani F. Clinical practice. Primary hyperparathyroidism. N Engl J Med. 2011;365(25):2389-2397.

  4. de FranciscolAL. Secondary hyperparathyroidism: review of the disease and its treatment. C in Ther. 2004;26(12):1976-1993.

  5. Jacobs TP, Bilezikian JP. Clinical review: rare causes of hypercalcemia. J Clin Endocrinol Metab. 2005;90(11):6316-6322.

  6. Orwoll ES. The milk-alkali syndrome: current concepts. Ann Intern Med. 1982;97(2):242- 248.

  7. Ross AC, Manson JE, Abrams SA, et al. The 2011 report on dietary reference intakes for calcium and vitamin D from the Institute of Medicine: what clinicians need to know. J

Clin Endocrinol Metab. 2011;96(1):53-58. 10. Penniston KL, Tanumihardjo SA. The acute and chronic toxic effects of vitamin A. Am J Clin Nutr. 2006;83(2):191-201. . Twigt BA, Houweling BM, Vriens MR, et al. Hypercalcemia in patients with bipola

11disorder treated with lithium: a cross-sectional study. Int J Bipolar Disord. 2013;1:18. r 12. Seccareccia D. Cancer-related hypercalcemia. Can Fam Physician. 2010;56(3):244-246, e90–e92.

  1. Sharma OP. Hypercalcemia in granulomatous disorders: a clinical review. Curr Opin Pulm Med. 2000;6(5):442-447.

  2. Kallas M, Green F, Hewison M, White C, Kline G. Rare causes of calcitriol-mediated hypercalcemia: a case report and literature review. J Clin Endocrinol Metab.

2010;95(7):3111-3117. 15. Winnacker JL, Becker KL, Katz S. Endocrine aspects of sarcoidosis. N Engl J Med. 1968;278(8):427-434.

  1. Liu JW, Huang TC,fLu YC, et al. Acute disseminated histoplasmosis complicated with hypercalcaemia. J In ect. 1999;39(1):88-90.
  2. Tuohy KA, Steinman TI. Hypercalcemia(due to excess 1,25-dihydroxyvitamin D in 18statement: diagnosis and management of beryllium sensitivity and chronic beryllium Crohn’s disease. Am J Kidney Dis. 2005;45 1):e3-e6. . Balmes JR, Abraham JL, Dweik RA, et al. An official American Thoracic Society disease. Am J Respir Crit Care Med. 2014;190(10):e34-e59.

  3. Chen K, Xie Y, Zhao L, Mo Z. Hyperthyroidism-associated:hypercalcemic crisis: a 20. Muls E,’Bouillon R, Boelaert J, et al. Etiology of hypercalcemia in a patient with case report and review of the literature. Medicine. 2017;96(4) e6017. Addison s disease. Calcif Tissue Int. 1982;34(6):523-526.

  4. Kannan CR. The Adrenal Gland. New York, NY: Plenum Publishing Corporation; 1988.
  5. Shah R, Licata A, Oyesiku NM, Ioachimescu AG. Acromegaly as a cause of 1,25-

207

dihydroxyvitamin D-dependent hypercalcemia: case reports and review of the literature. Pituitary. 2012;15 suppl 1:S17-S22. . Cano-Torres EA, Gonzalez-Cantu A, Hinojosa-Garza G, Castilleja-Leal F.

23Immobilization induced hypercalcemia. Clin Cases Miner Bone Metab. 2016;13(1):46-47. 24hypercalcemia in patientsfwith rhabdomyolysis with and without acute renal failure. J . Akmal M, Bishop JE, Tel er N, Norman AW, Massry SG. Hypocalcemia and Clin Endocrinol Metab. 1986;63(1):137-142.

  1. Rajkumar SV, Kumar S. Multiple myeloma: diagnosis and treatment. Mayo Clin Proc. 2016;91(1):101-119.

  2. Murray T, Long W,.Narins RG. Multiple myeloma and the anion gap. N Engl J Med. 27. Palumbo A, Anderson K. Multiple myeloma. N Engl J Med. 2011;364(11):1046-1060. 28. Hsu CC, Henry TS, Chung JH, Little BP. The incomplete border sign. J Thorac Imaging. 1975;292(11):574-575 2014;29(4):W48.

208

CHAPTER 10

209

Hypocalcemia

210 Case: An 18-year-old man with chest pain A previously healthy 18-year-old man presents to the emergency before arrival, he was playing basketball with his cousins when he developed numbness around his mouth and had difficulty freely department for evaluation of chest pain and other symptoms. Just complained of chest discomfort and difficulty breathing. He then move his hands and feet. The patient does not drink alcohol or use illicit substances. He is starting college soon and has been dealing with 38Heart rate is 120 beats per minute and regular, respiratory rate is lungs are cleartto auscultation. There are spasms of the hands and feet. ipSerum albumin is 4.2 g/dL, total serum calcium is 9.6 mg/dL (reference range 4.6-5.08 mg/dL). Arterial blood gas measurement the recent death of his father. breaths per minute, and blood pressure is 155/89 mm Hg. Hemoglobin oxygen saturation by pulse oximetry is normal on room air. The patien is in extremis, with Increased work of breathing. The Tapping the facial nerve elicits upward movement of the lips on the silateral side. (reference range 8.6-10.4 mg/dL), and ionized calcium is 3.1 mg/dL shows pH 7.71, partial pressure of carbon dioxide 18 mm Hg, and bicarbonate 22 mg/dL. Serum parathyroid hormone (PTH) is elevated. What is the most likely cause of hypocalcemia in this patient?

What is the normal Normal total serum calcium in adults is 8.6 to 10.4 mg/dL.1 range for total serum calcium concentration? How is calcium The vast majority of calcium (99%) is stored within bone, while 1% is found within intracellular fluid, and about distributed in the 0.1%, in extracellular fluid. ‹Total body calcium is determined by Gastrointestinal absorption and ‹renal excretion.2 body? How is calcium Under normal conditions, total serum calcium is divided between 3 forms: roughly 45% of total calcium is bound to distributed in plasma proteins (mostly albumin), 10% is bound to anions (eg, phosphate and citrate), and 45% is ionized blood? (unbound). Only ionized calcium is biologically active.1 What conditions Conditions that may change ionized calcium without changing total calcium concentration include alkalemia may result in a (decreases ionized fraction), acidemia (increases ionized fraction), and the presence of calcium chelators (decreases change in ionized ionized fraction). When these conditions are suspected, ionized calcium should be measured directly.1,3 calcium but not total serum calcium concentration? What conditions Conditions that may change total calcium without changing ionized calcium concentration include may result in a hypoalbuminemia (decreases total calcium), hyperalbuminemia (increases total calcium), and multiple myeloma change in total (increases total calcium).1-3 serum calcium but not ionized calcium concentration? How should the Corrected total serum calcium (g/dL) =measured total serum calcium (g/dL) +(0.8 ×[4 −serum album concentration measurement ofum (g/dL)]). When significant changes in serum protein levels are suspected, ionized calcium should be measured directly.1,3 total serum calci concentration be adjusted in the setting of hypoalbuminemia? What processes are Serum ionized calcium concentration is tightly regulated by the actions of PTH and activated vitamin D (ie, responsible for 1,25(OH) D, or 1,25-dihydroxyvitamin D). Hypocalcemia stimulates the secretion of PTH from the parathyroid 2 regulating serum glands and the production of activated vitamin D within the proximal tubule cells of the kidney. PTH acts directly at calcium concentration? 2 sites in the body to increase serum calcium: the bone (by stimulating osteoclasts) and the kidney (to increase proximal tubular reabsorption of calcium). By stimulating production of 1,25(OH)2D, PTH acts indirectly in the GI tract to increase calcium absorption. Through a negative feedback mechanism, PTH secretion from the parathyroid glands is turned off when calcium levels are normal or elevated (see Figure 9-2).2

211

Why is it important Hypocalcemia increases the excitability of nerve and muscle cells, which can lead to tetany; hypercalcemia decreases to maintain normal neuromuscular excitability, which can lead to cardiac dysrhythmias, lethargy, disorientation, and death. Clinical serum calcium manifestations associated with hypocalcemia increase in severity based on not only the degree of elevation in serum concentration? calcium, but also the rate at which it develops.1,4 What are the Mild and moderate hypocalcemia are commonly asymptomatic, depending on the rate of development and other clinical patient-specific factors. Hypocalcemia can lead to neuromuscular excitability with manifestations that may include manifestations of muscle twitching and spasm (eg, carpopedal spasm), tingling, numbness, hyperreflexia, tetany, seizure, QT hypocalcemia? prolongation, and cardiac dysrhythmia. Physical findings include Chvostek’s sign and Trousseau’s sign.4 What is Chvostek’s Chvostek’s sign describes facial muscle contraction on tapping the parotid gland over the facial nerve.4 sign? What is Trousseau’s sign describes the development of carpal spasms after inflating a sphygmomanometer cuff over the Trousseau’s sign? brachial artery above systolic blood pressure. It is more specific for hypocalcemia than Chvostek’s sign.4 What is the first Serum PTH level determines whether hypocalcemia is PTH-dependent (PTH level is low or normal) or PTH-step in determining independent (PTH level is elevated). the etiology of hypocalcemia?

In the setting of When serum calcium is low, PTH secretion from the parathyroid glands should increase in an attempt to return hypocalcemia, why serum calcium levels to normal. Therefore, a PTH level within the normal range is “inappropriately normal.” does a normal serum PTH value implicate a PTH- dependent process?

212

PTH-Dependent Hypocalcemia

What is the fundamental mechanism of PTH- dependent hypocalcemia? What general laboratory pattern is associated with hypoparathyroidism? The causes of hypoparathyroidism can be separated into which general subcategories?

PTH-dependent hypocalcemia occurs as a result of impaired secretion of PTH from the parathyroid glands (ie, hypoparathyroidism) despite low serum calcium levels. Hypoparathyroidism is generally associated with low serum calcium level, low or normal serum PTH level, and elevated serum phosphorus level. The causes of hypoparathyroidism can be separated into the following subcategories: iatrogenic, autoimmune, infiltrative, and other.

213

Iatrogenic Causes of Hypoparathyroidism

What are the iatrogenic causes of hypoparathyroidism?

You will find a transverse incision scar over the anterior neck. Nonsurgical therapy for head and neck cancers. Used in the treatment for ‹renally-induced secondary hyperparathyroidism.

Surgical removal. Ionizing radiation therapy. Calcimimetics.

What types of surgeries can lead to hypoparathyroidism?

What are the 2 scenarios in which the parathyroid glands may become damaged as a result of ionizing radiation therapy? What is the mechanism of PTH-dependent hypocalcemia related to cinacalcet?

Postsurgical hypoparathyroidism is the most common cause of hypoparathyroidism in adults. It typically results from the inadvertent or unavoidable removal of or damage to the parathyroid glands or the blood supply of the glands during surgeries such as thyroidectomy, parathyroidectomy, and radical neck dissection for head and neck cancer. Most cases are temporary, attributed to “stunning” of the glands, with patients regaining function within weeks to months after surgery. Chronic postsurgical hypoparathyroidism, defined as persistent hypoparathyroidism 6 months after surgery, is relatively rare.5 Iatrogenic hypoparathyroidism can occur following external beam radiation therapy for head and neck cancers, and radioactive iodine therapy for hyperthyroidism (rare).6

Cinacalcet, used in the treatment for renally-induced secondary hyperparathyroidism, decreases PTH secretion by activating the calcium-sensing receptors within the parathyroid gland. Patients with hypocalcemia should not receive cinacalcet.7

214

Autoimmune Causes of Hypoparathyroidism

What are the 2 general mechanisms of autoimmune-related hypoparathyroidism?

What are the characteristics of immune-mediated parathyroid gland destruction? Why does activation of the calcium-sensing receptor result in Decreased PTH secretion?

Hypoparathyroidism can occur via immune-mediated destruction of parathyroid tissue, or activation of the calcium-sensing receptor (CaSR) within the parathyroid glands.

Immune-mediated parathyroid destruction is the second most common cause of hypoparathyroidism in adults. It can occur in isolation or as part of a polyglandular syndrome. The vast majority of patients with polyglandular autoimmune syndrome type 1 have hypoparathyroidism; other main features include chronic mucocutaneous candidiasis and adrenal insufficiency.5 The CaSR is part of a negative feedback loop and is normally activated by elevated serum ionized calcium levels, which, in turn, decreases PTH release from the parathyroid glands, returning serum calcium levels to normal. Antibody-mediated activation of these receptors provides persistent negative feedback, resulting in inappropriately low PTH levels and hypocalcemia.5

215

Infiltrative Causes of Hypoparathyroidism

What are the infiltrative causes of hypoparathyroidism?

A 21-year-old man with a history of thalassemia, requiring multiple blood transfusions over his lifetime, is found to have hypocalcemia on routine laboratory evaluation. Characteristic apple-green color when tissue is stained with Congo red and viewed with polarized light. A common histopathologic finding of various infectious and autoimmune diseases.

Iron deposition.

Amyloidosis.

Granulomatous disease.

What are the characteristics of Hypoparathyroidism occurs most commonly as a result of secondary iron overload (eg, after multiple hypoparathyroidism related to blood transfusions in a patient with thalassemia) and only rarely as a result of primary iron infiltration? hemochromatosis. It occurs more frequently in male patients with peak prevalence around 20 years of age. It does not regress with iron chelation therapy. Hypoparathyroidism related to other heavy metals (eg, copper deposition in patients with Wilson’s disease) is exceedingly rare.6 What are the characteristics of Infiltration of the parathyroid gland with amyloid occurs in primary (AL) amyloidosis as well as hypoparathyroidism associated secondary (AA) amyloidosis, but it rarely causes a reduction in PTH production. More often, with amyloidosis? amyloidosis is associated with hypercalcemia as a result of its link to multiple myeloma.6 Which granulomatous diseases Sarcoidosis, tuberculosis, and syphilis are rare causes of hypoparathyroidism via gland infiltration. can cause hypoparathyroidism? More commonly, granulomatous diseases cause hypercalcemia via ectopic production of 1,25(OH) D.6 2 Infiltration of the parathyroid gland with malignancy (eg, metastatic disease or lymphoma) is a rare cause of hypoparathyroidism.

216

Other Causes of Hypoparathyroidism

What are the other causes of hypoparathyroidism?

Another electrolyte is to blame. A persistently stimulated negative feedback loop.

What is the mechanism of hypoparathyroidism related to hypomagnesemia? What is the mechanism of hypoparathyroidism related to hypermagnesemia? What are the clinical manifestations of activating mutations of the calcium- sensing receptor?

Magnesium derangement. Activating mutation of the calcium-sensing receptor.

The synthesis, release, and peripheral action of PTH are dependent on magnesium. Hypocalcemia related to magnesium deficiency is not responsive to exogenous administration of PTH, calcium, or vitamin D. The cornerstone of treatment is magnesium repletion.6

Hypermagnesemia inhibits PTH secretion. The hypoparathyroidism caused by hypermagnesemia is mild, typically asymptomatic, and reversible with normalization of magnesium levels.6

A variety of gain-of-function genetic mutations of the CaSR have been described. The degree of hypocalcemia and resultant clinical manifestations are variable, even between individuals with the same mutation. Most patients are asymptomatic until adulthood, but some present with severe manifestations of hypocalcemia, including seizures. Patients are at risk for nephrocalcinosis related to hypercalciuria, which occurs because the combination of low serum PTH and activation of the CaSR in the renal tubule decreases tubular reabsorption of calcium. This risk escalates when serum calcium levels increase from treatment, and such patients should be monitored closely.8

217

PTH-Independent Hypocalcemia

What is the fundamental mechanism of PTH- independent hypocalcemia? What general laboratory pattern is associated with PTH-dependent hypocalcemia? The causes of PTH-independent hypocalcemia can be separated into which general subcategories?

PTH-independent hypocalcemia occurs despite normal parathyroid gland function and Increased serum PTH levels. PTH-independent hypocalcemia is generally associated with low serum calcium level, elevated serum PTH level, and low serum phosphorus level. The causes of PTH-independent hypocalcemia can be separated into the following subcategories: vitamin D deficiency, consumption, and other.

218

PTH-Independent Hypocalcemia Related to Vitamin D Deficiency

What are the 2 main forms of biologically inactive vitamin D? What is the biologically active form of vitamin D? How can vitamin D deficiency be confirmed? What effect can chronic vitamin D deficiency have on bones?

Vitamin D2 (ie, ergocalciferol) and vitamin D3 (ie, cholecalciferol) are the main forms of inactive vitamin D (both forms are referred to as vitamin D). Sources of vitamin D include diet, dietary supplements, and sunlight exposure.2,9

1,25(OH)2D is biologically active. It primarily acts to increase Gastrointestinal calcium absorption, and calcium resorption from bone.2,9

The measurement of low serum 25-hydroxyvitamin D (ie, 25(OH)D) concentration confirms the diagnosis of vitamin D deficiency.1

Chronic vitamin D deficiency can result in deficient bone growth and mineralization. Rickets, which occurs in children, refers to deficient mineralization ‹at the growth plates and is characterized by various bone deformities ‹(Figure 10-1). Osteomalacia, which occurs in both children and adults, refers to impaired mineralization of the bone matrix and is characterized by pain, vertebral collapse, and fractures along stress lines.2,10

FIGURE 10-2 A young child with rickets. Note the bowed legs and thickening of the wrist and ankles. (From Becker KL, Bilezikian JP, Brenner WJ, et al. Principles and Practice ofiEndocrinology and Metabolism. 3rd ed. Philadelphia, PA: Lippincott Williams & Wilk ns; 2001.)

What are the causes of vitamin D deficiency?

This occurs during the many rainy months of Portland, Oregon. Jaundice, spider angiomas, and palmar erythema. Chronic anemia and shrunken kidneys on ultrasound. A 24-year-old man with Crohn’s disease presents with perioral numbness and is found to have hypocalcemia. A 42-year-old woman with a history of epilepsy develops hypocalcemia.

Sunlight deprivation.

Liver disease.

Chronic kidney disease.

Gastrointestinal malabsorption.

Antiepileptic medication.

219

What is the role of sunlight in vitamin D metabolism?

Ultraviolet B radiation induces the production of vitamin D3 from 7-dehydrocholesterol in the epidermis (Figure 10-2). At moderate to high latitudes, Decreased solar intensity and cold temperatures (particularly during the winter season) lead to reduced skin exposure to ultraviolet B radiation. In addition to sunlight deprivation, Increased skin pigmentation and skin thinning with age can contribute to vitamin D deficiency. In these populations, dietary sources of vitamin D become increasingly important.1,2,11

FIGURE 10-3 Metabolism of vitamin D and the regulation of serum calcium. (From Rubin R, Strayer DS. Rubin’s Pathology: Clinicopathologic Foundations of Medicine. 5th ed. Philadelphia, PA: Lippincott Williams & Wilkins; 2008.)

How is vitamin D The jejunum and ileum absorb vitamin D from dietary sources. Because it is fat soluble, absorption of vitamin D is absorbed in the facilitated by bile salts. Excess vitamin D is stored in adipose tissue and the liver, and can last for several months.2 Gastrointestinal tract? How does the liver Vitamin D and D are converted to 25(OH)D in the liver (see Figure 10-2). The liver is also a storage site of excess affect vitamin D 2 2 3 vitamin D. metabolism? How does the kidney The kidney converts 25(OH)D to the metabolically active 1,25(OH) D, ‹which then primarily acts in the 2 affect vitamin D Gastrointestinal tract and bone to increase serum calcium concentration (see Figure 10-2). Serum PTH, calcium, metabolism? and phosphorus levels regulate renal production of 1,25(OH)2D.2,9 What is the Various drugs accelerate the metabolism of vitamin D via activation of the xenobiotic receptor. Activation of the mechanism of vitamin xenobiotic receptor results in Increased synthesis of enzymes that degrade 25(OH)D and 1,25(OH) D. Common ‹ 2 D deficiency caused medications that act through this mechanism include phenytoin, carbamazepine, cyclophosphamide, by medication? dexamethasone, nifedipine, and spironolactone.9,12

220

PTH-Independent Hypocalcemia Related to Extravascular Consumption

What are the causes of PTH-independent hypocalcemia related to extravascular consumption?

Alcohol and gallstones are common causes of Acute pancreatitis. this condition. Consumed by another electrolyte. Hyperphosphatemia. A patient with dialysis-dependent renal Hungry bone syndrome. failure, complicated by secondary hyperparathyroidism with associated osteitis fibrosa cystica, undergoes parathyroidectomy and subsequently suffers from severe hypocalcemia for weeks despite elevated levels of serum PTH. A certain type of metastatic bone lesion. Osteoblastic bone metastases.

What are the characteristics of hypocalcemia related to acute pancreatitis?

What is the mechanism of hypocalcemia related to hyperphosphatemia?

What is hungry bone syndrome?

What are the characteristics of hypocalcemia related to osteoblastic bone lesions?

The mechanism of hypocalcemia in acute pancreatitis is thought to be related to autodigestion of mesenteric fat with release of free fatty acids, which consume calcium during the formation of calcium salts. Hypocalcemia is more likely to develop in severe cases of acute pancreatitis, and is a poor prognostic marker. Correction of hypocalcemia in patients with acute pancreatitis must be done cautiously as calcium plays a role in acinar cell injury and death.13 Hyperphosphatemia may lead to the precipitation of calcium phosphate in soft tissues, resulting in hypocalcemia. Acute hyperphosphatemia most commonly occurs as a result of cell breakdown (eg, tumor lysis syndrome, rhabdomyolysis) or Increased intake of phosphate (eg, Increased dietary intake, phosphate-containing enemas). Caution should be exercised in correcting hypocalcemia in the setting of hyperphosphatemia given the possibility of hastening widespread soft tissue calcium phosphate precipitation.14 Hungry bone syndrome describes rapid onset of severe and prolonged hypocalcemia following parathyroidectomy for severe hyperparathyroidism associated with high bone turnover. It usually lasts for 4 days or more, and is also characterized by hypophosphatemia and hypomagnesemia. The pathophysiology is thought to be Increased skeletal uptake of calcium following removal of high circulating levels of PTH. Hungry bone syndrome may last for months, requiring ongoing electrolyte monitoring and replacement.15 Hypocalcemia occurs in patients with osteoblastic bone lesions as a result of calcium consumption during formation of new bone around the metastatic lesions. Malignancies most commonly associated with osteoblastic metastases include prostate and breast cancer. Hypercalcemia is the most common calcium disturbance in patients with malignancy.16

221

PTH-Independent Hypocalcemia Related to Intravascular Consumption

What are the causes of PTH-independent hypocalcemia related to intravascular consumption?

Hypocalcemia develops after intense ‹vomiting in a patient with viral ‹gastroenteritis. Hypocalcemia develops in a trauma patient after receiving massive amounts of blood products. Used in the treatment for lead poisoning. A product of tissue ischemia. An antiviral medication.

What is the mechanism of hypocalcemia in the setting of alkalemia? What is the mechanism of hypocalcemia in the presence of citrate, EDTA, lactate, and foscarnet?

Alkalemia. Citrate. Ethylenediaminetetraacetic acid (EDTA). Lactate. Foscarnet.

Alkalemia enhances binding between albumin and calcium, thereby decreasing the ionized calcium levels (total calcium level remains unchanged).1,3 Citrate, EDTA, lactate, and foscarnet chelate calcium, thereby decreasing the ionized calcium level (total calcium level remains unchanged).17-19

222

Other Causes of PTH-Independent Hypocalcemia

What are the other causes of PTH-independent hypocalcemia?

Chronic alcohol consumption and Gastrointestinal malabsorption are common causes of this electrolyte disorder. Iatrogenesis. Genetic renal and bone insensitivity to PTH.

What is the mechanism of PTH-independent hypocalcemia related to hypomagnesemia? Which class of medication commonly used to treat hypercalcemia is associated with PTH-independent hypocalcemia? What general laboratory pattern is associated with pseudohypoparathyroidism?

Hypomagnesemia.

Medication. Pseudohypoparathyroidism.

Hypomagnesemia causes peripheral PTH resistance. The cornerstone of treatment for hypocalcemia related to magnesium deficiency is magnesium repletion.6 Bisphosphonates cause hypocalcemia as a result of potent inhibition of osteoclastic bone resorption. This effect is potentiated in patients with compromised renal function and in those with vitamin D deficiency.20 Pseudohypoparathyroidism describes various disorders defined by end-organ resistance to PTH. It is associated with low serum calcium and elevated serum PTH levels; unlike most causes of PTH-independent hypocalcemia, it is associated with hyperphosphatemia.5

223 Case Summary o An 18-year-old man with recent life stressors presents with acute- nset chest pain, perioral numbness, and carpopedal spasms and is found to have Decreased serum ionized calcium with normal total

serum calcium concentration. What is the most likely cause of hypocalcemia in this patient? Alkalemia.

224

Bonus Questions

Which specific acid-base The patient in this case has primary acute respiratory alkalosis. The history suggests acute-onset respiratory distress. Primary respiratory alkalosis isis only What is the most likely The patient in this case most likely experienced a panic attack related to life stressors that resulted in hyperventilation syndrome and respiratory alkalosis. disturbance is present in confirmed by the combination of alkalemia (arterial blood pH >7.45) and low partial pressure of carbon dioxide in arterial blood. Serum bicarbonate this case? mildly low, reflecting the expected degree of renal compensation for acute respiratory alkalosis. underlying condition in this case? What is hyperventilation Hyperventilation syndrome describes a constellationioflclinical manifestations that occur when minute ventilation exceeds metabolic needs. Episodes are often syndrome? triggered by stress. Symptoms of the syndrome may nclude dyspnea, light-headedness, numbness, paresthesias, and chest pain. These symptoms are the result of physiologic responses to hypocapnea and alka emia, including cerebral vasoconstriction, the Bohr effect, hypocalcemia, and hypophosphatemia. 21 What is the importance Normal total serum calcium levels may mask true hypocalcemia in certain circumstances, such as alkalemia or the presence of calcium chelators. Alkalemia ionized calcium of measuring serum promotes the binding of ionized calcium and albumin, thereby reducing serum ionized calcium without altering total serum calcium concentration. 1,3 concentration in this case? What is the treatment of Reassurance and counseling on behavioral modification is the cornerstone of management of hyperventilationfsyndrome. Patients may be more receptive toll choice for acute the diagnosis after provocation of symptoms with a “hyperventilation trial.” After breathing deeply at a rate o 30 to 40 breaths per minute, most patients wi hyperventilation experience recurrent symptoms within seconds to minutes. 21 syndrome? What are the principles Hypocalcemia is best managed by treating the underlying cause (eg, hyperventilation syndrome). Patients with hypocalcemia complicated by neuromuscular of the management of irritability require hospitalization and treatment with intravenous calcium. Asymptomatic patients with corrected total serum calcium <7.6 mg/dL may hypocalcemia? develop serious complications and hospitalization should be considered. Intravenous calcium gluconate is preferred over calcium chloride because it is less likely to cause local irritation at the infusion site. 4

225 Key Points

Serum calcium levels are tightly regulated by the actions of PTH and the active form of vitamin D. Hypocalcemia can lead to neuromuscular excitability with

manifestations that include muscle twitching and spasm, tingling, numbness, hyperreflexia, tetany, seizure, QT prolongation, and cardiac dysrhythmia.

Clinical manifestations associated with hypocalcemia increase in severity based on not only the degree of elevation of serum calcium, but also the rate at which it develops.

Physical findings of hypocalcemia include Chvostek’s sign and Serum PTH level determines whether hypocalcemia is PTH- Trousseau’s sign. dependent or PTH-independent. The causes of PTH-dependent hypocalcemia (ie, hypoparathyroidism) can be separated nto the following

subcategories: iatrogenic, autoimmune,iinfiltrative, and other. d Patients with,symptomatic hypocalcemia should be hospitalized The long-term management of hypocalcemia depends on the The causes of PTH-independent hypocalcemia can be separate into the following subcategories: vitamin D deficiency, consumption and other. and treated with intravenous calcium. underlying cause.

226

References 1. Minisola S, Pepe J, Piemonte S,:Cipriani C. The diagnosis and management of 2. Berne RML, Levy MN. Physiology. 4th ed. St. Louis, MO: Mosby, Inc.; 1998. hypercalcaemia. BMJ. 2015;350 h2723. 2. Stewart AF. Clinical practice. Hypercalcemia associated with cancer. N Engl J Med. 2005;352(4):373-379.

  1. Cooper MS, Gittoes NJ. Diagnosis and management of hypocalcaemia. BMJ. 2008;336(7656):1298-1302. Bilezikian JP, Khan A, Potts JT Jr, et al. Hypoparathyroidism in the adult:

  2. epidemiology, diagnosis, pathophysiology, target-organ involvement, treatment, and challenges for future research. J Bone Miner Res. 2011;26(10):2317-2337.

  3. Brandi ML, Brown EM, eds. Hypoparathyroidism. Milan, Italy: Springer-Verlag Italia; 2015.

  4. Poon G. Cinacalcet hydrochloride (Sensipar). Bayl Univ Med Cent Proc. 2005;18(2):181- 184. Lienhardt A, Bai M, Lagarde JP, et al. Activating mutations of the calcium-sensing

  5. receptor: management of hypocalcemia. J Clin Endocrinol Metab. 2001;86(11):5313-5323. 9. Holick MF. Vitamin D deficiency. N Engl J Med. 2007;357(3):266-281.

  6. Sahay M, Sahay:R. Rickets-vitamin D deficiency and dependency. Indian J Endocrinol 11. Engelsen O. The relationship between ultraviolet radiation exposure and vitamin D Metab. 2012;16(2) 164-176. status. Nutrients. 2010;2(5):482-495.

  7. Grober U, Kisters K. Influence of drugs on vitamin D and calcium metabolism. Dermatoendocrinol. 2012;4(2):158-166.

  8. Ahmed A, Azim A, Gurjar M, Baronia AK. Hypocalcemia in acute pancreatitis revisited. Indian J Crit Care Med. 2016;20(3):173-177.

  9. Sutters M,iGaboury CL, Bennett WM. Severe hyperphosphatemia and hypocalcemia: 15challenge in the post-operative management of primary hyperparathyroidism: al a a dilemma n patient management. J Am Soc Nephrol. 1996;7(10):2056-2061. . Witteveen JE, van Thiel S, Romijn JA, Hamdy NA. Hungry bone syndrome: stil systematic review of the literature. Eur J Endocrinol. 2013;168(3):R45-R53. . Kassi E, Kapsali I, Kokkinos M, Gogas H. Treatment of severe hypocalcaemia due to

16osteoblastic metastases in a patient with post-thyroidectomy hypoparathyroidism with 153Sm-EDTMP. BMJ Case Rep. 2017;2017. . Cairns CB, Niemann JT Pelikan PC, Sharma J Ionized hypocalcemia during

17prolonged)cardiac arrest,and closed-chest CPR.in a canine model. Ann Emerg Med. 18trauma patients receiving massive transfusion. J Surg Res. 2016;202(1):182-187.in 19induced hypocalcemia and effects of foscarnet on calcium metabolism. J Clin Endocrinol 1991;20(11 :1178-1182. . Giancarelli A, Birrer KL, Alban RF, Hobbs BP, Liu-DeRyke X. Hypocalcemia . Jacobson MA, Gambertoglio JG, Aweeka FT, Causey DM, Portale AA. Foscarnet- Metab. 1991;72(5):1130-1135.

  1. Do WS, Park JK, Park MI, Kim HS, Kim SH, Lee DH. Bisphosphonate-induced severe hypocalcemia—a case report. J Bone Metab. 2012;19(2):139-145.

  2. Magarian GJ. Hyperventilation syndromes: infrequently recognized common expressions of anxiety and stress. Medicine. 1982;61(4):219-236.

227

228

CHAPTER 11

229

Hypothyroidism

230 Case: A 38-year-old woman with bradycardia A previously healthy 38-year-old woman is brought to the hospital complained of fatigue, weight gain, and constipation for several episodes of drowsiness, forgetfulness, and inability to concentrate that drowsier on the day of presentation, prompting her husband to bring p Temperature is 35.3°C, and heart rate is 46 beats per minute. The extremities. There are 5 vitiliginous patches on the trunk. Reflexes are r for confusion. Her husband provides most of the history. She has months. Over the past few weeks, the patient’s husband has noticed have been occurring with increasing frequency. She was much her in for evaluation. atient is somnolent but arouses when prompted. She is easily distracted and oriented to self but not place or time. A photo of the patient is shown in Figure 11-1. There is nonpitting edema of the lowe symmetric with a delayed relaxation phase. Laboratory evaluation is notable for serum sodium of 118 mEq/L, 4.2 mIU/L), and free thyroxine of 0.12 ng/dL (reference range 0.6- thyroid stimulating hormone of 192 mIU/L (reference range 0.4- 1.2 ng/dL).

FIGURE 11-1

What is the most likely diagnosis in this patient?

What is hypothyroidism? What is the normal hormonal cycle of the hypothalamic-pituitary- thyroid axis?

Hypothyroidism is a clinical condition that results from thyroid hormone deficiency. The hypothalamus produces thyroid-releasing hormone (TRH), which stimulates the pituitary to secrete thyroid-stimulating hormone (TSH). The thyroid gland responds to TSH stimulation by secreting thyroxine (T4) and triiodothyronine (T3), which then provide negative feedback to both the hypothalamus and pituitary (Figure 11-2).

231

FIGURE 11-2 Schematic of the hypothalamic-pituitary-thyroid axis. Regulatory feedback relationships are designated with arrows.

What is the relationship between T3 and T4?

How common is hypothyroidism?

What are the symptoms of hypothyroidism?

What are the physical findings of hypothyroidism?

What life-threatening complication can occur in patients with untreated hypothyroidism? If hypothyroidism is suspected clinically, what is the next diagnostic step?

T3 and T4 are both produced in the thyroid gland by the follicular cells, although the vast majority of thyroid output is in the form of T4. Peripheral conversion of T4 to T3 occurs in the liver and kidney (see Figure 11-2). The relative potency of T3 is several times greater than that of T4, and it is more biologically active.1 The prevalence of hypothyroidism in the general population varies by sex and age. It is more common in women with an overall prevalence of up to 5% in the industrialized world. Incidence in women rises with age, particularly after 45 years.2,3 Symptoms of hypothyroidism depend on the age and sex of the patient but in general may include weight gain, fatigue, poor concentration, depression, constipation, cold intolerance, dry skin, proximal muscle weakness, hair thinning or loss, and menorrhagia.4 Physical findings of hypothyroidism depend on the age and sex of the patient but in general may include hypothermia, bradycardia, diastolic hypertension, cognitive impairment, coarse facies, lateral eyebrow thinning (ie, Queen Anne’s sign), periorbital edema, goiter (see Figure 12-2), dry or coarse skin, hoarse voice, delayed relaxation phase of deep tendon reflexes, nonpitting peripheral edema, and macroglossia.4 Myxedema coma is a severe and life-threatening complication of hypothyroidism, usually associated with a precipitating factor such as infection. It is more common in older patients, particularly women. Patients present with mental status changes and other severe manifestations of hypothyroidism, such as lethargy, cognitive dysfunction and psychosis, hypothermia, bradycardia, hyponatremia, and hypoventilation. Myxedema coma requires prompt recognition and treatment.5 In patients with a clinical condition compatible with hypothyroidism, low serum free T4 will confirm the diagnosis. Concurrent measurement of serum TSH level can distinguish whether the process is TSH- independent (TSH is elevated) or TSH-dependent (TSH is low or normal).

In the setting of When there is thyroid hormone deficiency, pituitary TSH secretion should increase in an attempt to return hypothyroidism, why serum thyroid hormone levels to normal. Therefore, a TSH level within the normal range is “inappropriately does a normal serum normal.” TSH value implicate a TSH-dependent process? Which coexisting Pregnancy, nonthyroid acute illness, medications (eg, glucocorticoids), and recovery from thyrotoxicosis can conditions can make the make the interpretation of thyroid function studies challenging.6 interpretation of thyroid function studies difficult? What clinical entity is Subclinical hypothyroidism is defined by the presence of normal serum free T with elevated serum TSH. This 4 characterized by the diagnosis is valid only when such results have been demonstrated over the course of at least several weeks, the combination of normal serum free T4 and hypothalamic-pituitary-thyroid axis is normal, and there is no recent or ongoing nonthyroidal illness.7 elevated serum TSH? How common is Subclinical hypothyroidism is present in up to 8% of the general population in the industrialized world.7 subclinical hypothyroidism? In patients with In women with subclinical hypothyroidism, the risk of developing overt hypothyroidism is 2% to 3% per year, subclinical and this rises to 4% per year when serum antithyroid peroxidase antibodies are present. The relative risk of hypothyroidism, what is progression is even higher in men, but the overall prevalence of overt hypothyroidism in men remains the risk of developing significantly lower than in women.7 overt hypothyroidism

232

over time? Should patients with subclinical hypothyroidism be treated with thyroid hormone? Why is it helpful to separate the causes of hypothyroidism into TSH-independent and TSH-dependent processes?

What is the relative prevalence of primary and central hypothyroidism?

It remains unclear whether there are benefits to treating patients with subclinical hypothyroidism with thyroid hormone. Treatment should be considered for symptomatic patients and in those who are pregnant or attempting to conceive.4,7

TSH-independent hypothyroidism indicates intrinsic dysfunction of the thyroid gland (ie, primary hypothyroidism), whereas TSH-dependent hypothyroidism suggests dysfunction in the hypothalamic or pituitary components of the regulatory axis (ie, central hypothyroidism).

Central hypothyroidism is rare, accounting for <1% of all cases, and estimated to be 1000 times less common than primary hypothyroidism.2,8

233

Primary Hypothyroidism

What is the fundamental mechanism of primary hypothyroidism?

Primary hypothyroidism occurs as a result of failure of the thyroid gland to produce adequate thyroid hormones despite stimulation from increased TSH.

The causes of primary hypothyroidism can be separated into which general subcategories?

The causes of primary hypothyroidism can be separated into the following subcategories: thyroiditis, iodine-related, iatrogenic, and infiltrative.

234

Primary Hypothyroidism Related to Thyroiditis

What is thyroiditis?

During the phase of thyrotoxicosis in patients with thyroiditis, what is the typical result of a radioactive iodine uptake test?

Thyroiditis refers to a group of disorders that produce inflammation of the thyroid gland, which may lead to the destruction of follicular cells (thyrocytes) and unregulated release of preformed thyroid hormones into the circulation. There is typically a phase of excess circulating thyroid hormone (ie, thyrotoxicosis) followed by either normal thyroid function or hypothyroidism, which may be temporary or permanent.9,10 Thyroiditis is generally associated with decreased radioiodine uptake (see Figure 12-3). The thyrotoxic phase of Hashimoto’s thyroiditis (ie, Hashitoxicosis) can be associated with normal or even increased radioactive iodine uptake.9,11

What causes of thyroiditis can lead to hypothyroidism?

Responsible for the vast majority of cases of primary hypothyroidism in iodine-sufficient parts of the world; this condition is associated with other autoimmune diseases, such as vitiligo, pernicious anemia, celiac disease, autoimmune adrenalitis, and type 1 diabetes mellitus. Two months after giving birth, a 29-year-old woman experiences a period of restlessness, tremor, and weight loss, followed by weight gain, constipation, and dry skin. A 50-year-old woman presents during the summer with anterior neck pain after a recent upper respiratory tract infection and is found to have a painful goiter on examination. Iatrogenic causes of thyroiditis. Mild symptoms of hyperthyroidism followed by transient hypothyroidism in a patient with a nontender thyroid gland. A 62-year-old woman with rheumatoid arthritis treated with immunosuppressive medications presents with left anterior neck pain and is found to have an exquisitely tender thyroid gland with overlying erythema, fever, and positive blood cultures.

What serologic studies are supportive of the diagnosis of Hashimoto’s thyroiditis?

What are the characteristics of postpartum thyroiditis?

What are the characteristics of subacute thyroiditis?

What medications can cause thyroiditis?

What are the 2 main types of radiation--

Hashimoto’s thyroiditis.2

Postpartum thyroiditis.

Subacute thyroiditis (ie, de Quervain’s thyroiditis).

Drug-induced thyroiditis and radiation-induced thyroiditis. Painless thyroiditis (ie, silent thyroiditis).

Infectious thyroiditis.

Thyroid autoantibodies (eg, antithyroid peroxidase antibodies and antithyroglobulin antibodies) are present in the majority of patients with Hashimoto’s thyroiditis. Unlike most other causes of thyroiditis, Hashimoto’s thyroiditis virtually always causes permanent hypothyroidism.2 Postpartum thyroiditis occurs during the first postpartum year with a mean prevalence of 10%. It is a manifestation of an underlying but previously clinically silent autoimmune thyroiditis, which becomes unmasked by “immunologic rebound” after pregnancy. The most common presentation is isolated hypothyroidism, but it can also present with transient hyperthyroidism with or without subsequent hypothyroidism. The majority of patients are euthyroid within 1 year postpartum; however, permanent hypothyroidism develops in some.12 Subacute thyroiditis is most likely caused by viral infection and commonly presents with a tender thyroid gland and associated systemic manifestations. Symptoms typically last for weeks to months but can be improved with nonsteroidal anti-inflammatory drugs along with glucocorticoids in severe cases. The majority of patients fully recover, but some develop permanent hypothyroidism.13 Medications associated with thyroiditis include amiodarone, interferon-α, interleukin-2, lithium, and tyrosine kinase inhibitors. Medication-induced thyroiditis is usually painless.14 Patients treated with radioactive iodine therapy for hyperthyroidism rarely develop

235

What are the characteristics of painless thyroiditis?

What are the risk factors for developing infectious thyroiditis?

after exposure. Thyroiditis may also be caused by external beam radiation therapy for head and neck cancers. Risk.factors include high-dose therapy, young age, female sex, and preexisting hypothyroidism 14 Painless thyroiditis is an autoimmune condition similar to postpartum thyroiditis, although the affected populations include men, and women who are not within the peripartum period. It typically presents with transient hyperthyroidism, which can be followed by thyroid recovery or hypothyroidism. The majority of patients who experience hypothyroidism recover completely after 3 months, but up to one-fifth will develop permanent hypothyroidism.15 Infectious thyroiditis is a rare condition but occurs more frequently in patients with certain congenital abnormalities (eg, persistent thyroglossal duct), those who are older in age, and those who are immunocompromised. Pathogens include bacteria (eg, Streptococcus pyogenes, Staphylococcus aureus, Streptococcus pneumoniae), fungi, and parasites.14

236

Primary Hypothyroidism Related to Iodine

What is the role of iodine in thyroid physiology?

Iodine is an essential element of the T4 and T3 hormones, and most of the body’s iodine stores are contained within the thyroid gland. Iodide, the reduced form of iodine, modulates thyroid function and is capable of decreasing thyroid hormone production and release. Both iodine deficiency and excess can lead to thyroid dysfunction.16

What are the iodine-related causes of hypothyroidism?

Dietary fortification has decreased the burden of this condition, but it remains prevalent in some parts of the world, including Africa and Asia. A 34-year-old woman with subclinical hypothyroidism develops symptoms and signs of hypothyroidism after undergoing an imaging study with iodinated intravenous contrast.

How common is hypothyroidism caused by iodine deficiency? What are the characteristics of iodine-induced hypothyroidism?

Iodine deficiency.2

Iodine excess (Wolff-Chaikoff effect).

Iodine deficiency is the most common cause of hypothyroidism in the developing world. Patients commonly develop a large goiter as a result of TSH-mediated adaptation to iodine deficiency.7 Sources of excess iodine include diet, medications (eg, amiodarone), and iatrogenic administration of a radiocontrast agent. Iodine excess can activate an autoregulatory phenomenon known as the Wolff-Chaikoff effect that inhibits iodination of thyroglobulin to prevent thyrotoxicosis. Usually, this effect only lasts for a few days, but in some individuals, the hypothyroid state persists. Patients at highest risk include the elderly, those with underlying thyroid disease (eg, subclinical hypothyroidism), and those with chronic nonthyroidal illness (eg, cystic fibrosis). Thyroid function usually returns to baseline within 2 to 8 weeks of iodine withdrawal (although cases of amiodarone-induced hypothyroidism may be prolonged because of the long half-life of the drug).17

237

Primary Hypothyroidism Related to Iatrogenesis

What are the iatrogenic causes of hypothyroidism?

Definitive treatment for Thyroidectomy or radioactive iodine ablation. hyperthyroidism. A 46-year-old man with bipolar Lithium. disorder develops weight gain, cold intolerance, and hair loss a few months after starting a mood- stabilizing medication. Look for tattoo marks over the External beam radiation therapy. anterior neck.

How common is permanent hypothyroidism after thyroidectomy?

How common is permanent hypothyroidism after radioactive iodine therapy for Graves’ disease? How common is hypothyroidism caused by lithium?

What are the characteristics of hypothyroidism related to external beam radiation of the neck?

Hypothyroidism is inevitable after total thyroidectomy but also occurs in up to one-third of patients who undergo hemithyroidectomy. Risk factors include preexisting thyroid disease such as subclinical hypothyroidism or Hashimoto’s thyroiditis. Hypothyroidism begins 2 to 4 weeks after total thyroidectomy; in cases of hemithyroidectomy, it can develop months or years following the procedure.18 The development of hypothyroidism after radioactive iodine ablation is dose-dependent, but the majority of patients develop hypothyroidism within the first year. After the first year, the incidence is 2% to 3% per year.19 Lithium interferes with thyroid hormone secretion, causing an increase in TSH and hypertrophy of thyroid tissue. Long-term lithium therapy results in a goiter in up to one-half of patients, subclinical hypothyroidism in up to one-third, and overt hypothyroidism in up to 15%. Other medications that can cause hypothyroidism include tyrosine kinase inhibitors, interferon-α, thalidomides, and antiepileptic drugs (eg, valproate).2,20 Hypothyroidism is the most common late clinical sequela of external beam radiation to the neck. It is dose-dependent and occurs more commonly in women and in patients who undergo combined neck surgery and radiation. Patients can develop subclinical or overt hypothyroidism; patients who develop subclinical hypothyroidism progress to overt hypothyroidism at a high rate over time.21

238

Primary Hypothyroidism Related to Infiltrative Disorders

What are the infiltrative causes of hypothyroidism?

Patients who have received numerous blood transfusions, such as those with thalassemia, are at risk for developing this condition. Characterized by the accumulation of amorphous, proteinaceous material. Noncaseating granulomas on histologic evaluation. A fibrotic condition originally described with the German term “eisenharte,” which means iron-hard, to denote the fixed hard enlargement of the thyroid gland.

How common is hypothyroidism in patients with hemochromatosis?

How common is hypothyroidism in patients with amyloidosis?

How common is hypothyroidism in patients with sarcoidosis?

What imaging finding can be helpful in distinguishing Riedel’s thyroiditis from other infiltrative disorders? What other infiltrative disorders can involve the thyroid gland and result in hypothyroidism?

Hemochromatosis.

Amyloidosis. Sarcoidosis. Riedel’s thyroiditis.22

Endocrine disorders are common in patients with hemochromatosis; the most frequent disorders are diabetes mellitus and hypogonadism. Hypothyroidism occurs in 10% of men with hemochromatosis. Iron loss through menstruation is protective against iron accumulation in women with hemochromatosis, which delays the onset of clinical sequelae.23 Clinically significant thyroid involvement (ie, amyloid goiter) is uncommon in patients with systemic amyloidosis. However, patients who do develop amyloid goiter are frequently hypothyroid. Treatment is aimed at the underlying condition.24 Thyroid infiltration from sarcoidosis is rare. Although there is evidence of thyroid involvement on autopsy in 4% of patients with sarcoidosis, it is clinically significant in <1% of cases.25 The extension of fibrotic changes on imaging beyond the thyroid capsule to adjacent structures, such as encasement of the carotid artery, can be a clue to Riedel’s thyroiditis.22 Hypothyroidism from infiltration of the thyroid gland can occur in patients with scleroderma, lymphoma, leukemia, and cystinosis.26

239

Central Hypothyroidism

What is the fundamental mechanism of central hypothyroidism? What are some clues to the presence of central hypothyroidism? Which glands are involved in central hypothyroidism?

What general processes can cause pituitary and/or hypothalamic dysfunction? What is the most common cause of central hypothyroidism?

Central hypothyroidism occurs as a result of understimulation of the thyroid gland due to inadequate production of TSH.

In patients presenting with the typical manifestations of hypothyroidism, a central process is more likely in those with preexisting hypothalamic or pituitary disease, history of head trauma, headaches, visual field defects, and focal neurologic findings.8 Central hypothyroidism can occur as a result of pituitary gland dysfunction (ie, secondary hypothyroidism) or hypothalamic dysfunction (ie, tertiary hypothyroidism).

Pituitary and/or hypothalamic dysfunction can be caused by medication (eg, glucocorticoids), mass lesion (eg, brain metastasis), traumatic brain injury, subarachnoid haemorrhage, infection/abscess, stroke, external beam radiation, pituitary apoplexy, Sheehan’s syndrome, autoimmune disease (eg, lymphocytic hypophysitis), and infiltrative disease (eg, hemochromatosis).2,8 Pituitary macroadenomas (eg, prolactinoma) are responsible for more than half of cases of central hypothyroidism.8

240 Case Summary A 38-year-old woman with recent weight gain and constipation reflexes, hyponatremia, lowiserum free T4, and elevated serumiTSH. presents with confusion and is found to have hypothermia, bradycardia, nonpitting per pheral edema, abnormal neurolog c What is the most likely diagnosis in this patient? Primary hypothyroidism.

241

Bonus Questions

What physical finding oft the The photograph of the.patient in this case (see Figure 11-1) shows thinning of the eyebrows, particularly the lateral third (known as the Sign of Hertoghe photograph of the patien in this case? hypothyroidism is seen in or Queen Anne’s sign) What is the significance of The patient in this case has TSH-independent (primary) hypothyroidism, which is characterized by the combination of elevated serum TSH and depressed the thyroid biochemical serum free T . In TSH-dependent (central) hypothyroidism, serum TSH would be low or normal in the setting of low serum free T . profile in this case? 4 4 What typeiof primaryst Hashimoto’s thyroiditis causes the vast majority of cases of primary hypothyroidism in the industrialized world and is statistically most likely in this case. likely in this case? hypothyro dism is mo The presence of serum thyroid autoantibodies would support the diagnosis. 2 How should this patient be The patient in this case has myxedema coma based on the presence of somnolence and delirium. The cornerstone of therapy is immediate thyroidhe 4 3 managed? hormone replacement. Synthetic T may be given alone or in combination with T . Intravenous administration is recommended initially because t presence of bowel edema may limit absorption. In cases of secondary hypothyroidism or polyglandular autoimmune syndrome, baseline cortisol should always be obtained to evaluate for coexistent adrenal insufficiency. If present, glucocorticoids should be given before thyroid replacement. What is the initial Patients with permanent hypothyroidism should be started on oral synthetic thyroxine (ie, levothyroxine) according to body mass. A daily dose of 5

management of permanent 1.6 ”g/kg body mass (equivalent to 100 ”g daily for the average-sized woman)iis-sufficient for most adults. Older patients (>60 years of age) or those with present? hypothyroidism when ischemic heart disease should be started on a lower dose of levothyroxine (eg, 25 50 ”g daily) and titrated up gradually. For optimal and consistent myxedema coma is not absorption, levothyroxine should be taken on an empty stomach, usually 60 m nutes before breakfast or at bedtime (at least 3 hours after dinner). 27,28 After starting replacement After starting therapy for primary hypothyroidism, serum TSH and free T levels should be measured after 4 to 6 weeks, with adjustments made to the 4 hormone, how should patients with primary dose accordingly. Serum TSH should be rechecked in 4 to 6 week intervals after any dose adjustment. The “goal” serum TSH level is within the lower half of the reference range. Once a dose is found that achieves a serum TSH level within the goal, it can be measured annually. Dose adjustments may need to hypothyroidism be monitored? be considered sooner for weight changes or pregnancy.27 How soon do patients with The half-life ofilevothyroxine is 7 days, so it takes at least a week for the symptoms of hypothyroidism to begin to improve. Muscle weakness and hormone replacement has improvement after thyroid hypothyroidism notice an cognitive defic ts may take up to 6 months to fully recover. 28 been started?

242 Key Points

Hypothyroidism is a clinical condition that results from thyroid Hypothyroidism is more common in women than men. concentration, depression, constipation, coldlintolerance, dry skin, hormone deficiency. Symptoms of hypothyroidism include weight gain, fatigue, poor proximal muscle weakness, hair thinning or oss, and menorrhagia. Physical findings of hypothyroidism include hypothermia,

bradycardia, diastolic hypertension, cognitive impairment, coarse voice, delayed relaxation phase of deep tendon reflexes, nonpitting hypothyroidism, usually associated with a precipitating factorof When hypothyroidism is suspected clinically, it is confirmed with Serum TSH level determines whether hypothyroidism is TSH- facies, lateral eyebrow thinning, goiter, dry or coarse skin, hoarse peripheral edema, and macroglossia. Myxedema coma is a severe and life-threatening complication such as infection. 4 a low serum free T . independent (primary) or TSH-dependent (central). The causes of primary hypothyroidism can be separated into the

following subcategories: thyroiditis, iodine-related, iatrogenic, and infiltrative. The most common causes of primary hypothyroidism are

Hashimoto’s thyroiditis (in the industrialized world) and iodine Central hypothyroidism is rare and caused by pituitary and/or hypothyroidism; mosttadults require a daily dose of 1.6 ”g/kg deficiency (in the developing world). hypothalamic dysfunction. Synthetic thyroxine is he treatment of choice for primary body mass.

243

References 1. Berne RML, Levy MN. Physiology. 4th ed. St. Louis, MO: Mosby, Inc.; 1998. 2. Chaker L, Bianco AC, Jonklaas J, Peeters RP. Hypothyroidism. Lancet. 2017;390(10101):1550-62.

  1. Tunbridge WM, Evered DC, Hall R, et al. The spectrum of thyroid disease in a community: the Whickham survey. Clin Endocrinol. 1977;7(6):481-493.

  2. Gaitonde DY, Rowley KD, Sweeney LB. Hypothyroidism: an update. Am Fam Physician. 2012;86(3):244-251.

  3. Mathew V, Misgar RA,:Ghosh S, et al. Myxedema coma: a new look into an old crisis. J 6. Koulouri O, Moran C, HalsallfD, Chatterjee K, Gurnell M. Pitfalls in the measurement Thyroid Res. 2011;2011 493462. and interpretation of thyroid unction tests. Best Pract Res Clin Endocrinol Metab. 2013;27(6):745-762.

  4. Garber JR, Cobin RH, Gharib H, et al. Clinical practice guidelines for hypothyroidism in adults: cosponsored by the American Association of Clinical Endocrinologists and

the American Thyroid Association. Endocr Pract. 2012;18(6):988-1028. 8. Persani L.iClinical review: central hypothyroidism: pathogenic, diagnostic, and therapeut c challenges. J Clin Endocrinol Metab. 2012;97(9):‹3068-3078. 9. De Leo S, Lee SY, Braverman LE. Hyperthyroidism. Lancet. 2016;388(10047):906-918. 10. Pearce EN, Farwell AP, Braverman LE. Thyroiditis. N Engl J Med. 2003;348(26):2646- 2655.

  1. Intenzo CM, CapuzziiDM, Jabbour S, Kim SM, dePapp AE. Scintigraphic features of autoimmune thyroidit s. Radiographics. 2001;21(4):957-964.
  2. Stagnaro-Green)A. Clinical.review 152: postpartum thyroiditis. J Clin Endocrinol Metab. 2002;87(9 :4042-4047 . Fatourechi V, Aniszewski JP, Fatourechi GZ, Atkinson EJ, Jacobsen SJ. Clinical 13features and outcome of subacute thyroiditis in an incidence cohort: olmsted County, Minnesota, study. J Clin Endocrinol Metab. 2003;88(5):2100-2105.

  3. Bindra A, Braunstein GD. Thyroiditis. Am Fam Physician. 2006;73(10):1769-1776. 15. Mittra ES, McDougall IR. Recurrent silent thyroiditis: a report of four patients and review of the literature. Thyroid. 2007;17(7):671-675.

  4. Chung HR. Iodine and thyroid function. Ann Pediatr Endocrinol Metab. 2014;19(1):8- 12.

  5. Markou K, Georgopoulos N, Kyriazopoulou V, Vagenakis AG. Iodine-Induced hypothyroidism. Thyroid. 2001;11(5):501-510.

  6. Stoll SJ, Pitt SC, Liu J, Schaefer S, Sippel RS, Chen H. Thyroid hormone replacement after thyroid lobectomy. Surgery. 2009;146(4):554-558; discussion 8-60.

  7. Mumtaz M, Lin LS, Hui KC, Mohd Khir AS. Radioiodine I-131 for the therapy of graves’ disease. Malays J Med Sci. 2009;16(1):25-33.

  8. George J, Joshi SR. Drugs and thyroid. J Assoc Physicians India. 2007;55:215-223. 21radiation-induced hypothyroidism: a literature-based,meta-analysis. Cancer.for . Vogelius IR, Bentzen SM, Maraldo MV, Petersen PM Specht L. Risk factors 2011;117(23):5250-5260.

  9. Hennessey;JV. Clinical review: Riedel’s thyroiditis: a clinical review. J Clin Endocrinol Metab. 2011 96(10):3031-3041.

244

  1. Edwards CQ, Kelly TM, Ellwein G, Kushner JP. Thyroid disease in hemochromatosis. Increased incidence in homozygous men. Arch Intern Med. 1983;143(10):1890-1893.

  2. Kimura H, Yamashita S, Ashizawa K, Yokoyama N, Nagataki S. Thyroid dysfunction in patients with amyloid goitre. Clin Endocrinol. 1997;46(6):769-774. . Kmiec P, Lewandowska M, Dubaniewicz A, et al. Two cases of hyroid sarcoidosi

25presentation as painful, recurrent goiter in patients with Graves’tdisease. Arq Brass Endocrinol Metabol. 2012;56(3):209-214.

  1. Braverman LE, Cooper DS, eds. Werner and Ingbar’s the Thyroid: A Fundamental and Clinical Text. 10th ed. Philadelphia: Lippincott Williams & Wilkins; 2013. . Jonklaas J, Bianco AC, Bauer AJ, et al. Guidelines for the treatment of

27hypothyroidism: prepared by the american thyroid association task force on thyroid hormone replacement. Thyroid. 2014;24(12):1670-1751.

  1. Vaidya B, Pearce SH. Management of hypothyroidism in adults. BMJ. 2008;337:a801.

245

CHAPTER 12

246

Thyrotoxicosis

247 Case: A 64-year-old woman with palpitations several weeks of intermittent palpitations. The episodestseem to occur A previously healthy 64-year-old woman presents to he clinic with randomly, including when she is at rest. When symptomatic, she does not have chest pain but does feel short of breath. She has had an

unintentional weight loss of 10 pounds over this time. Her husband Heart rate is 120 beats per minute and irregular, and blood pressure reports she has seemed more irritable lately. is 150/70 mm Hg. There is a fine tremor of the hands. The eyes appear to bulge anteriorly. The tissue over the anterior neck is generous in f appearance, and a continuous bruit is heard when the diaphragm o the stethoscope is placed over it. Serum thyroid-stimulating hormone is measured at 0.01 mIU/L

(reference range 0.4-4.2 mIU/L).iFree thyroxine is 6.8 ng/dLt‹(reference range 0.6-1.2 ng/dL). A radioact ve iodine uptake test with hyroid scan shows diffusely Increased uptake (Figure 12-1).

FIGURE 12-1 (Courtesy of Mary H. Samuels, MD.)

What is the most likely diagnosis in this patient?

What is thyrotoxicosis?

What is the normal cycle of the hypothalamic- pituitary-thyroid axis?

What is the relationship between T3 and T4?

How common is thyrotoxicosis? What are the symptoms of thyrotoxicosis?

What are the physical findings of thyrotoxicosis?

Thyrotoxicosis is a clinical condition that results from excess circulating thyroid hormones from any source. Hyperthyroidism specifically refers to Increased thyroid hormone synthesis from the thyroid gland.1 The hypothalamus produces thyroid-releasing hormone (TRH), which stimulates the pituitary to secrete thyroid-stimulating hormone (TSH). The thyroid gland responds to TSH stimulation by secreting thyroxine (T4) and triiodothyronine (T3), which then provide negative feedback to both the hypothalamus and pituitary (see Figure 11-2).2 T3 and T4 are both produced in the thyroid gland by the follicular cells, although the vast majority of thyroid output is in the form of T4. Peripheral conversion of T4 to T3 occurs in the liver and kidney (see Figure 11-2). The relative potency of T3 is several times greater than that of T4, and it is more biologically active.2 In the industrialized world, thyrotoxicosis affects 1 in 2000 persons per year.3

Symptoms of thyrotoxicosis depend on underlying cause, duration of disease, and patient-specific factors such as age and sex but in general may include heat intolerance, sweating, nervousness, irritability, anxiety, fatigue, poor concentration, palpitations, dyspnea, hyperdefecation, nausea, vomiting, menstrual irregularities, diplopia, and eye discomfort.1 Physical findings of thyrotoxicosis depend on the underlying cause, duration of disease, and patient-specific factors such as age and sex but in general may include weight loss, tremor of the extremities, hyperreflexia, tachycardia, systolic hypertension, tachypnea, goiter (with or without bruit), abdominal tenderness, pelvic and shoulder muscle weakness, warm and moist skin, exophthalmos (ie, proptosis) (Figure 12-2), eyelid retraction and lag, periorbital edema, and ophthalmoplegia.1

248

FIGURE 12-2 A young woman with hyperthyroidism. Note the anterior neck mass (goiter) and bulging of the

How does the presentation of thyrotoxicosis vary with age? What life-threatening complication can occur in patients with untreated thyrotoxicosis? If thyrotoxicosis is suspected based on the clinical evaluation, what is the next diagnostic step?

In the setting of thyrotoxicosis, why does a normal serum TSH value implicate a TSH- dependent process? Which coexisting conditions can make the interpretation of thyroid function studies difficult? What condition is characterized by the combination of normal serum thyroid hormone levels and suppressed TSH? What condition is characterized by the combination of normal serum free T4, suppressed TSH, and elevated total T3 or free T3?

eyes (exophthalmos). (Reprinted with permission from Rubin E, Farber JL. Pathology. 3rd ed. Philadelphia, PA: Lippincott Williams & Wilkins; 1999.)

Younger patients with thyrotoxicosis are more likely to have hyperadrenergic manifestations (eg, anxiety, restlessness, tremor); older patients tend to be less symptomatic but are more likely to develop cardiovascular complications (eg, dysrhythmia).1 Thyroid storm describes a severe and life-threatening form of thyrotoxicosis with extreme signs and symptoms of hypermetabolism, including tachycardia, fever, perspiration, diarrhea, anxiety, seizure, delirium, and coma. There is often a precipitating factor, such as discontinuation of antithyroid drugs, infection, trauma, stress, or pregnancy. The distinction is clinical, as thyroid hormone levels are comparable to those in patients with compensated thyrotoxicosis. The condition is life-threatening; up to one-quarter of patients die.1 In patients with a clinical condition compatible with thyrotoxicosis, elevated serum free T4 will confirm the diagnosis. Concurrent measurement of serum TSH can distinguish whether the process is TSH-independent (TSH level is low) or TSH-dependent (TSH level is elevated or normal).

When there is thyroid hormone excess, negative feedback to the hypothalamus and pituitary should turn off TSH secretion in an attempt to return serum thyroid hormone levels to normal. Therefore, a TSH level within the normal range is “inappropriately normal.”

Pregnancy, nonthyroid acute illness, medications (eg, glucocorticoids), and recovery from thyrotoxicosis can make the interpretation of thyroid function studies challenging.4

Subclinical hyperthyroidism is defined by the presence of low serum TSH with normal serum levels of thyroid hormone. Over time, patients with subclinical hyperthyroidism may remain stable, progress to overt thyrotoxicosis, or revert to euthyroid state. Risk of progression is Increased when serum TSH levels are <0.1 mIU/L. There is an Increased risk of atrial fibrillation in patients with subclinical hyperthyroidism. The decision to treat is controversial and depends on the patient’s age and degree of TSH suppression.5 T3 toxicosis describes the combination of normal serum free T4, suppressed TSH, and elevated total T3 or free T3. This pattern can be seen early in the course of hyperthyroidism. It also occurs with a higher frequency in patients with the combination of hyperthyroidism (eg, Graves’ disease) and concurrent iodine deficiency.6,7

249

TSH-Independent Thyrotoxicosis

What is the fundamental mechanism of TSH-independent thyrotoxicosis? Once TSH-independent thyrotoxicosis is established, what is the next diagnostic step?

TSH-independent thyrotoxicosis occurs as a result of the presence of excess thyroid hormones independent of TSH stimulation of the thyroid gland.

Patients with TSH-independent thyrotoxicosis should undergo a radioactive iodine uptake test. Patients are given radioactive iodine and then a Îł probe is used to detect how much of it is taken up by the thyroid gland. Uptake can be normal, Increased, or Decreased. True hyperthyroidism (ie, Increased thyroid hormone synthesis from the thyroid gland) results in Increased radioactive iodine uptake, whereas thyrotoxicosis without hyperthyroidism results in Decreased uptake.

250

TSH-Independent Thyrotoxicosis with Increased Radioactive Iodine Uptake

What are the 2 general patterns of Increased radioactive iodine uptake?

When radioactive iodine uptake is Increased, a diffuse or focal pattern of uptake can be identified by thyroid scintigraphy (ie, thyroid scan) (Figure 12-3). Thyroid scintigraphy is different than, but adjunctive to, the radioactive iodine uptake test.

FIGURE 12-3 An illustration depicting the patterns of radioactive iodine uptake by thyroid scan in patients with various forms of thyrotoxicosis. One important exception is the thyrotoxic phase of Hashimoto’s thyroiditis (ie, Hashitoxicosis), which can be associated with normal or even Increased radioactive iodine uptake. (From Chowdhury SH, Cozma AI, Chowdhury JH. Essentials for the Canadian Medical Licensing Exam. 2nd ed. Philadelphia, PA: Wolters Kluwer Health; 2017.)

251

TSH-Independent Thyrotoxicosis with Diffusely Increased Radioactive Iodine Uptake

What are the causes of TSH-independent thyrotoxicosis with diffusely Increased radioactive iodine uptake?

Associated with TSH- Graves’ disease. receptor antibodies, this condition is the most common cause of thyrotoxicosis in iodine-sufficient parts of the world. A hormone produced Human chorionic gonadotropin (hCG). during pregnancy with a subunit that is identical to that of TSH.

What are the characteristics of Graves’ disease?

What are the sources of hCG-induced hyperthyroidism?

Graves’ disease accounts for the majority of cases of thyrotoxicosis in iodine-sufficient parts of the world. Its cause is multifactorial but involves the development of autoantibodies that activate the TSH receptor and stimulate thyroid follicular cells. There is a higher prevalence of Graves’ disease in women. Clinical manifestations specific to Graves’ disease include Graves’ ophthalmopathy, including exophthalmos (see Figure 12-2), pretibial myxedema (ie, thyroid dermopathy) (see Figure 23-6), and a systolic or continuous bruit over the thyroid gland.1,8 Because of its structural similarity to TSH, hCG can stimulate thyroid TSH receptors, causing hyperthyroidism. The most common causes of hCG-induced hyperthyroidism include pregnancy (typically multiple gestation) and gestational trophoblastic tumors.1

252

TSH-Independent Thyrotoxicosis with Focally Increased Radioactive Iodine Uptake

What are the causes of TSH-independent thyrotoxicosis with focally Increased radioactive iodine uptake?

Multiple focal areas of uptake. A “hot nodule.”

What are the characteristics of toxic multinodular goiter?

Do all thyroid adenomas result in thyrotoxicosis?

Toxic multinodular goiter (TMNG).

Toxic adenoma.

TMNG is the most common cause of thyrotoxicosis in iodine-deficient parts of the world and is more frequent in women and the elderly. In the setting of iodine deficiency, chronic TSH stimulation results in the growth of thyroid tissue that can produce thyroid hormone autonomously, independent from stimulation by TSH. This usually results in suppression of the surrounding normal thyroid tissue. Patients typically develop TMNG after a long history of euthyroid multinodular goiter. Thyrotoxicosis occurs when the autonomous nodules “tip the balance” toward excess thyroid hormone. TMNG is best treated with radioactive iodine ablation or surgery, as antithyroid drugs alone are unlikely to achieve remission.1,9 Thyroid adenomas are present in around one-half of the general population at autopsy, although the majority are never diagnosed. Adenomas typically present as palpable nodules discovered by the patient or physician but are also discovered incidentally during imaging studies. The vast majority of adenomas are clinically silent or “cold.” Only 5% of thyroid nodules are functional or “hot” and capable of causing thyrotoxicosis (see Figure 12-3). The majority of all thyroid nodules are benign. However, malignancy is more likely in patients with a thyroid nodule when serum TSH is not suppressed. Such cold nodules should be evaluated with biopsy. Although far less likely, malignancy can occur in patients with functional adenomas. Thyrotoxicosis related to thyroid adenoma is best treated with radioactive iodine ablation or surgery, as antithyroid drugs alone are unlikely to achieve remission.1,9,10

253

TSH-Independent Thyrotoxicosis with Decreased Radioactive Iodine Uptake

What are the causes of TSH-independent thyrotoxicosis with Decreased radioactive iodine uptake?

A 25-year-old woman with a history of anorexia nervosa presents with symptoms and signs of thyrotoxicosis. Typically characterized by a phase of thyrotoxicosis followed by either normal thyroid function or hypothyroidism, which may be temporary or permanent. Jod-Basedow phenomenon. The thyroid gland is an innocent bystander.

For what reason might a young woman take thyroid medication surreptitiously? What is thyroiditis?

What is the Jod-Basedow phenomenon?

What source of extraglandular thyroid hormone production occurs only in women?

Exposure to exogenous thyroid hormone.

Thyroiditis.

Iodine exposure. Extraglandular production of thyroid hormone.

Surreptitious thyroid hormone ingestion, known as thyrotoxicosis factitia, is often used to augment weight loss. The finding of undetectable serum thyroglobulin levels can distinguish thyrotoxicosis factitia from other causes of TSH-independent thyrotoxicosis. However, the presence of thyroglobulin antibodies can interfere with the assay, producing misleading results. In such cases, Increased fecal thyroxine levels suggest the diagnosis.11,12 Thyroiditis refers to a group of disorders that produce inflammation of the thyroid gland, which may lead to destruction of thyrocytes and the unregulated release of preformed thyroid hormones into the circulation. There are several distinct clinical entities that cause painful thyroiditis, such as subacute thyroiditis (ie, de Quervain’s thyroiditis), infectious thyroiditis (ie, suppurative thyroiditis), radiation-induced thyroiditis, and trauma-induced thyroiditis. Those that are not painful include Hashimoto’s thyroiditis, painless thyroiditis (ie, silent thyroiditis), postpartum thyroiditis, drug-induced thyroiditis, and fibrous thyroiditis (ie, Riedel’s thyroiditis). The thyrotoxic phase of Hashimoto’s thyroiditis (ie, Hashitoxicosis) can be associated with normal or even Increased radioactive iodine uptake.1,13,14 The Jod-Basedow phenomenon refers to iodine-induced hyperthyroidism. It typically occurs in patients with preexisting autonomous thyroid tissue related to iodine deficiency (eg, multinodular goiter). When the iodine supply is Increased, the autonomous regions begin to increase thyroid hormone synthesis independent of autoregulatory mechanisms, resulting in thyrotoxicosis within weeks to months. Sources of iodine include radiographic contrast media and medications (eg, amiodarone). It is typically self-limited once the source of excess iodine has been removed (cases of amiodarone-induced hyperthyroidism may be particularly prolonged because of the long half-life of the drug).15 Extraglandular thyroid hormone production is very rare. Struma ovarii describes an ovarian tumor that contains functioning thyroid tissue1

254

255

TSH-Dependent Thyrotoxicosis

What is the fundamental mechanism of TSH-dependent thyrotoxicosis? What general term is used to describe thyrotoxicosis related to excess TSH?

TSH-dependent thyrotoxicosis occurs as a result of excess TSH stimulation of the thyroid gland that responds by producing excess thyroid hormones. Thyrotoxicosis related to excess TSH is referred to as TSH-induced hyperthyroidism.

What are the causes of TSH-induced hyperthyroidism?

A 55-year-old man Pituitary adenoma. presents with weight loss, headache, and tremor and is found to have a visual field defect on examination. The hypothalamus Impaired sensitivity to thyroid hormone. and pituitary do not respond to negative feedback.

What are the characteristics of TSH-induced hyperthyroidism caused by pituitary adenoma? What is impaired sensitivity to thyroid hormone?

TSH-secreting pituitary adenomas are rare, representing 0.5% to 3% of all pituitary adenomas. Both genders are equally affected, and most patients are diagnosed in the fifth to sixth decades of life. The clinical manifestations tend to be mild compared with other forms of thyrotoxicosis. Some tumors secrete both TSH and growth hormone, giving rise to a mixed clinical picture of thyrotoxicosis and acromegaly. The majority of TSH-secreting pituitary adenomas are large and invasive, with associated headache, visual field defects, and loss of vision. Magnetic resonance imaging of the brain should be performed in patients suspected of having a TSH-secreting pituitary adenoma.16 Impaired sensitivity to thyroid hormone describes any condition that reduces the effectiveness of thyroid hormone. Resistance to thyroid hormone is the most common subtype and usually occurs as a result of an inherited mutation in the thyroid hormone receptor. The degrees of thyroid hormone resistance and TSH-mediated compensation vary between patients, but there are also considerable differences within each individual’s own peripheral tissues, giving rise to a unique mixture of clinical features of hyperthyroidism and hypothyroidism. The most common manifestations of thyrotoxicosis in these patients include goiter, sinus tachycardia, and hyperactivity.17,18

256 Case Summary A 64-year-old woman presents with palpitations, weight loss, and

dyspnea and is found to have irregularly irregular tachycardia, wide diffusely Increased radioiodine uptake as measured by a radioactive 4 pulse pressure, elevated serum free T , suppressed serum TSH, and iodine uptake test and thyroid scan.

What is the most likely diagnosis in this patient? Graves’ disease.

257 Bonus Questions

What heart The irregularly irregular tachycardia described in this case is most likely atrial fibrillation, which occurs at a higher rate in patients with thyrotoxicosis.1 likely present in rhythm is most this case? Which features in Weight loss, irritability,lpalpitations, and tremor may be features of thyrotoxicosis fromiany cause. However, the exophthalmos (bulgingtof the eyes anteriorly) and this case are bruit over the thyroid g and (systolic or continuous in nature) described in this case are specific for Graves’ disease. Another finding specific to Graves’ disease (not specifically present in this case) is pretibial myxedema (ie, thyroid dermopathy), characterized by p gmented and thickened skin, most evident over he shins (see Figure 23- 6 suggestive of ). 1,8 Graves’ disease? What causes thy? In some patientstwith Graves’ disease, autoimmune-mediated lymphocytic infiltration of the orbital tissues (including adipose tissue and the extraocular muscles) l What measurables Antibodiesiagainst the TSH receptorrare present in the serum of patients with Graves’ disease and can be measured to confirm the diagnosis when radioactive iodine Graves’ leads to deposition of glycosaminoglycans, resulting in edema and Increased orbital volume. Over time, this process leads to fibrosis and scarring within the orbita ophthalmopa tissues. The mos common manifestations include exophthalmos, periorbital edema, and diplopia. 1,19 present in patient with Graves’ autoantibody is uptake test ng is unavailable or cont aindicated. 1 disease? What class ofn be Thyrotoxicosis leads to an increase in ÎČ-adrenergictreceptors in peripheral tissues, resulting in manifestations such as anxiety, tachycardia, and palpitations. ÎČ- used to treat the hyperadrenergic medication ca Blockers can be used to ameliorate these manifesta ions. 20,21

manifestations of thyrotoxicosis? What options are Options for long-term treatment of Graves’ disease generally include antithyroid drugs (eg, thionamides), radioactive iodine ablation, and surgery. Treatmen available for long- approach depends on regional practice, patient-specific factors (eg, age, severity of symptoms), and patient preference. Several modalities may ultimately be t term treatment of necessary to achieve remission. 1 hyperthyroidism ’ caused by Graves disease? How effective a The thionamide agents (eg, methimazole) treat hyperthyroidism by inhibiting the synthesis of thyroid hormones. These drugs are effec ive in achieving remission i the antithyroidre patients with Graves’ disease, but relapse rates are high when discontinued, particularly within the first year.-The risk fortrecurrence istgreater in those with severen 4 3 drugs? hyperthyroidism, large goiter, high T :T ratios, persistently suppressed TSH, and high baseline titers of TSH receptor an ibodies. In patients with toxic adenomas and TMNG, remission is rarely achieved with antithyroid drugs alone; however, these agents can be useful in combination with definitive therapy (eg, radioactive iodine ablation or surgery). The most common major side effect of the thionamides is agranulocytosis, which can be life-threatening. 1 When can Radioactive iodine therapy can be used as first-line treatment in patients with Graves’ disease, toxic adenoma, and TMNG. Absolute contraindications include therapy be used to with this condition. The choice between radioactive iodine therapy and thyroidectomy depends on regional practice, patient-specific factors, and patient preference.1 hyperthyroidism? radioactive iodine pregnancy, breastfeeding, and planned pregnancy. Radioactive iodine therapy can make Graves’ ophthalmopathy worse and should be avoided in most patients treat When can Thyroidectomy can be used to treat Graves’ disease, toxic adenoma, and TMNG. Thyroidectomy is generally recommended in patients with large goiters, low thyroidectomy be uptake of radioactive iodine, suspected or documented thyroid cancer, and moderate to severe ophthalmopathy. Major adverse effects include hypothyroidism, used to treat hypoparathyroidism, and recurrent laryngeal nerve injury. 1 hyperthyroidism?

258 Key Points

Thyrotoxicosis is a clinical condition that results from thyroid nervousness, irritability, anxiety, fatigue, poor concentration,g, hormone excess. Symptoms of thyrotoxicosis include heat intolerance, sweatin palpitations, dyspnea, hyperdefecation, nausea, vomiting, menstrual irregularities, diplopia, and eye discomfort. Physical findings of thyrotoxicosis include weight oss, tremor of

the extremities, hyperreflexia, tachycardia, systoliclhypertension, tachypnea, goiter with or without bruit, abdominal tenderness, pelvic and shoulder muscle weakness, warm and moist skin,

exophthalmos, eyelid retraction and lag, periorbital edema, and ophthalmoplegia. Younger patients with thyrotoxicosis are more likely to have

hyperadrenergic manifestations (eg, anxiety, restlessness, tremor); older patients tend to be less symptomatic but are more likely to develop cardiovascular complications (eg, dysrhythmia).

Thyroid storm describes a severe and life-threatening form of thyrotoxicosis with extreme signs and symptoms of hypermetabolism.

When thyrotoxicosis is suspected clinically, it is confirmed with an Serum TSH level determines whether thyrotoxicosis is TSH- 4 elevated serum free T . independent or TSH-dependent. TSH-independent thyrotoxicosis can be associated with Increased Increased radioactive iodine uptake can occur in.a diffuse or focal Optionstfor the long-term treatment for hyperthyroidism related to generally include antithyroid drugs (eg, thionamides), radioactive underlying cause, regional practice, patient-specific factors, and or Decreased radioactive iodide uptake. pattern, which informs the differential diagnosis ÎČ-Blockers can be used to ameliorate the hyperadrenergic manifes ations of thyrotoxicosis (eg, anxiety, tachycardia). Graves’ disease, toxic adenoma, and toxic multinodular goiter iodine ablation, and/or surgery. Choice of modality depends on patient preference.

259

260

References 1. De Leo S, Lee SY, Braverman LE. Hyperthyroidism. Lancet. 2016;388(10047):906-918. 2. Berne RML, Levy MN. Physiology. 4th ed. St. Louis, Missouri: Mosby, Inc.; 1998. 2. Vaidya B, Pearce SH. Diagnosis and management of thyrotoxicosis. BMJ. 2014;349:g5128.

  1. Koulouri O, Moran C, HalsallfD, Chatterjee K, Gurnell M. Pitfalls in the measurement and interpretation of thyroid unction tests. Best Pract Res Clin Endocrinol Metab. 2013;27(6):745-762. Biondi B, Bartalena L, Cooper DS, Hegedus L Laurberg P, Kahaly GJ. The 2015

  2. European Thyroid Association Guidelines on,diagnosis and treatment of endogenous subclinical hyperthyroidism. Eur Thyroid J. 2015;4(3):149-163. Figge J, Leinung M, Goodman AD, et al. The clinical evalua ion of patients with

  3. subclinical hyperthyroidism and free triiodothyronine (freetT3) toxicosis. Am J Med. 1994;96(3):229-234.

  4. Hollander CS, Mitsuma T, Shenkman L, Stevenson C, Pineda G, Silva E. T3 toxicosis in an iodine-deficient. Lancet. 1972;2(7790):1276-1278.

  5. Williams E, Chillag.S, Rizvi A. Thyroid bruit and the underlying ‘inferno’. Am J Med. 9. Krohn K, FuhreriD, Bayer Y, et al. Molecular pathogenesis of euthyroid and toxic 10hyperfunctioning thyroid nodule harboring thyroid carcinoma: review of the 2014;127(6):489-490 multinodular go ter. Endocr Rev. 2005;26(4):504-524. . Mirfakhraee S, Mathews D, Peng L, Woodruff S, Zigman JM. A solitary literature. Thyroid Res. 2013;6(1):7. . Bouillon R, Verresen L, Staels F, Bex M, De Vos P, De Roo M. The measurement of

11fecal thyroxine in the diagnosis of thyrotoxicosis factitia. Thyroid. 1993;3(2):101-103. 12. Mariotti S, Martino E, CupinitC, et al. Low serum thyroglobulin as a clue to the diagnosis of thyrotoxicosis fac itia. N Engl J Med. 1982;307(7):410-412. 13. Pearce EN, Farwell AP, Braverman LE. Thyroiditis. N Engl J Med. 2003;348(26):2646- 2655.

  1. Intenzo CM, CapuzziiDM, Jabbour S, Kim SM, dePapp AE. Scintigraphic features of 15Basedow syndrome following oral iodine and radioiodinated-antibodyiadministration. autoimmune thyroidit s. Radiographics. 2001;21(4):957-964. . El-Shirbiny AM, Stavrou SS, Dnistrian A, Sonenberg M, Larson SM, D vgi CR. Jod- J Nucl Med. 1997;38(11):1816-1817.

  2. Beck-Peccoz P, Lania A, Beckers A, Chatterjee K, Wemeau JL. 2013 European thyroid association guidelines for the diagnosis and treatment of thyrotropin-secreting

pituitary tumors. Eur Thyroid J. 2013;2(2):76-82. 17. De Groot LJ, Chrousos G, Dungan K, et al, eds. Endotext. South Dartmouth, MA: MDText.com, Inc.; 2000.

  1. Refetoff S, DeWind LT, DeGroot LJ. Familial syndrome combining deaf-mutism, stuppled epiphyses, goiter and abnormally high PBI: possible target organ

refractoriness to thyroid hormone. J Clin Endocrinol Metab. 1967;27(2):279-294. 19. Maheshwari R, Weis E. Thyroid associated orbitopathy. Indian J Ophthalmol. 2012;60(2):87-93. . Bilezikian JP, Loeb JN. The influence of hyperthyroidism and hypothyroidism on

20alpha- and beta-adrenergic receptor systems and adrenergic responsiveness. Endocr

261

Rev. 1983;4(4):378-388. 21. Geffner DL, Hershman JM. Beta-adrenergic blockade for the treatment of hyperthyroidism. Am J Med. 1992;93(1):61-68.